SlideShare a Scribd company logo
1 of 76
Interpreter Ethics: Professional Role
Boundaries for Healthcare Interpreters
Eliana Lobo & Jazmin Manjarrez
OSTI 2017
Training objectives
Be able to describe:
 what a code of ethics is
 why a code of ethics exists for every health profession
 what the interpreter code of ethics covers
Be able to explain what you would do in specific situations, such as:
 what to do when you are asked to perform beyond your scope of practice
 patient calls you at home to get your medical advice, and wants a ride to next appointment
 care team asks you to report what patient said to you in waiting room
 the family with a new baby tucks a gift in your briefcase
 you made a mistake and you realize it a few minutes later
 your sister runs a store with ethnic remedies that you think would work for the patient
 you interpret for a person that you strongly disapprove of based on personal choices
 you could make a lot of money interpreting for your extended family and close friends 2
What is a code of ethics?
 A code is a set of rules
 Everyone within a profession or function is supposed to
follow this set of rules
 A code is usually set by the people who will live by it and
whose reputation it defines
 A code of ethics describes correct conduct in a certain role
3
Health Care Code of Ethics
(in general) Doctors, Nurses, Pharmacists, Therapists...
 Protects patients from harm
 correct info for good health outcomes
 financial safety from predators
 maintains privacy and empowers patient
 Clarifies the role of the worker
 scope, what does he do, what does he not do
 professional boundaries and relationships
 Defines a superior worker from those not so good
 quality of work
 professional habits
4
Code of Ethics--Quiz
A code of ethics provides guidelines for:
A) rate of pay for workers
B) role boundaries of the worker vis-a-vis clients
C) how a worker can achieve a higher level within the
hierarchy of his trade
D) behavior of the worker towards his family
5
National Codes of Ethics
for Medical Interpreters
 NCIHC
http://www.ncihc.org/ethics-and-standards-of-practice
 IMIA
http://www.imiaweb.org/code/
6
NCIHC - Code of Ethics for Interpreters in Health Care
7
1. ‰The interpreter treats as confidential, within the treating team, all information learned in the performance of
their professional duties, while observing relevant requirements regarding disclosure
2. ‰The interpreter strives to render the message accurately, conveying the content and spirit of the original
message taking into consideration its cultural context
3. The interpreter maintains the boundaries of the professional role, refraining from personal involvement
4. ‰‰The interpreter continuously strives to develop awareness of his/her own and other (including biomedical) cultures
encountered in the performance of their professional duties
5. ‰The interpreter strives to maintain impartiality & refrains from counseling, advising or projecting personal biases or beliefs
6. ‰‰When the patient’s health, well-being, or dignity is at risk, the interpreter may be justified in acting as an advocate.
-Advocacy is understood as an action taken on behalf of an individual that goes beyond facilitating communication,
with the intention of supporting good health outcomes.
-Advocacy must only be undertaken after careful and thoughtful analysis of the situation and if other less intrusive actions
have not resolved the problem
7. The interpreter treats all parties with respect
8. ‰The interpreter strives to continually further his/her knowledge and skills
9. ‰The interpreter must at all times act in a professional and ethical manner
IMIA – Code of Ethics
8
The most important of these principles are:
1. Interpreters will maintain confidentiality of all assignment-related information
2. Interpreters will select the language and mode of interpretation that most accurately conveys the content and spirit of the
messages of their clients
3. Interpreters will refrain from accepting assignments beyond their professional skills, language fluency, or level of training
4. Interpreters will refrain from accepting an assignment when family or close personal relationships affect impartiality
5. Interpreters will not interject personal opinions or counsel patients
6. Interpreters will not engage in interpretations that relate to issues outside the provision of health care services unless
qualified to do so
7. Interpreters will engage in patient advocacy and in the intercultural mediation role of explaining cultural differences, or practices
to health care providers /patients only when appropriate and necessary for communication purposes, using professional judgment
8. Interpreters will use skillful unobtrusive interventions so as not to interfere with the flow of communication in triadic medical settings
9. Interpreters will keep abreast of their evolving languages and medical terminology
10. Interpreters will participate in continuing education programs as available
11. Interpreters will seek to maintain ties with relevant professional organizations in order to be up-to-date with the latest
professional standards and protocols
12. Interpreters will refrain from using their position to gain favors from clients.
State Codes of Ethics
for Medical Interpreters
California – CHIA
 http://c.ymcdn.com/sites/www.chiaonline.org/resource/resmgr/
docs/standards_chia.pdf
Oregon - NCIHC
 http://ostiweb.org/wp-content/uploads/2016/03/Interpreting-
codes-of-ethics-that-apply-in-Oregon.pdf
Washington - DSHS
 https://www.dshs.wa.gov/fsa/language-testing-and-certification-
program/code-ethics
9
CA Endowment & CHIA –Ethical Principles
10
Each ethical principles is to be considered in the context of the health and well-being of the patient.
1. Confidentiality Interpreters treat all information learned during the interpreting as confidential.
2. Impartiality Interpreters are aware of the need to identify any potential or actual 10 Executive
Summary conflicts of interest, as well as any personal judgments, values, beliefs or opinions that
may lead to preferential behavior or bias affecting the quality and accuracy of the interpreting
performance.
3. Respect for individuals and their communities Interpreters strive to support mutually respectful
relationships between all three parties in the interaction (patient, provider and interpreter), while
supporting the health and well-being of the patient as the highest priority of all healthcare
professionals.
4. Professionalism and integrity Interpreters conduct themselves in a manner consistent with
the professional standards and ethical principles of the healthcare interpreting
profession. 5. Accuracy and completeness Interpreters transmit the content, spirit and cultural
context of the original message into the target language, making it possible for patient and provider
to communicate effectively.
5. Cultural responsiveness Interpreters seek to understand how diversity and cultural similarities and
differences have a fundamental impact on the healthcare encounter. Interpreters play a critical role
in identifying cultural issues and considering how and when to move to a cultural
clarifier r ole. Developing cultural sensitivity and cultural responsiveness is a life-long process
that begins with an introspective look at oneself.
Oregon Code of Ethics
for Healthcare Interpreters
Code of Ethics and Standards of Practice for Interpreters in Health Care
1. Health care interpreters must adhere to the National Code
of Ethics for Interpreters in Health Care as established by
the National Council on Interpreting in Health Care.
2. Healthcare interpreters must adhere to the National
Standards of Practice for Interpreters in Health Care as
established by the National Council on Interpreting in
Health Care.
11
WA DSHS–Code of Ethics for Medical Interpreters
12
Accuracy. Interpreters/translators must always express the source language message in a thorough and faithful
manner. They must: omit/add nothing; Give consideration to linguistic variations in both the source and target languages;
Conserve the tone and spirit of the source language.
Cultural sensitivity-courtesy. Interpreters/translators must be culturally sensitive, & respectful of whom they serve.
Confidentiality. Interpreters/translators must not divulge any information publicly or privately obtained through their
assignments, including, but not limited to, information gained through access to documents or other written materials.
Proficiency. Interpreters/translators must meet the minimum proficiency standard set by DSHS.
Compensation. Interpreters/translators must: Not accept additional money, consideration, or favors for services reimbursed
by the department. The fee schedule agreed to between the contracted language services providers and the department shall
be the maximum compensation accepted.
Nondiscrimination. Interpreters/translators must: Always be impartial and unbiased; Not discriminate on the basis of gender,
disability, race, color, national origin, age, socio-economic or educational or marital status, religious or political beliefs, or
sexual orientation; and Refuse or withdraw from an assignment, without threat or retaliation, if they are unable to perform the
required service in an ethical manner.
Self-representation. Interpreters/translators must accurately and completely represent their certifications, training, and
experience.
Impartiality-conflict of interest. Interpreters/translators must disclose to the department any real or perceived conflicts of
interest that would affect their professional objectivity.
Professional demeanor. Interpreters/translators must be punctual, prepared, and dressed in a manner
appropriate, and not distracting for the situation.
Scope of practice. Interpreters/translators must not: Counsel, refer, give advice, or express personal
opinions to the individuals for whom they are interpreting/translating;
Pair & Share Exercise
Turn to your neighbor and share an ethical quandary or consideration
from your work experience, or from a personal observation
13
Interpreter Code of Ethics
Why did we fight so hard to create it?
True patient stories:
• “My interpreter told me that the other interpreter in town would gossip about me, so I
should always get appointments that would fit with his schedule. This slowed down my
course of treatment so much that I lost my insurance before I could finish treatment,
and now I can’t afford to go to the doctor.”
• “My interpreter was with me when I had to terminate my pregnancy due to very severe
abnormalities in the fetus. The interpreter keeps telling me that God does not allow
abortions. I am so depressed that I cannot take care of myself or my two children.”
• “My interpreter took a picture of me and posted it on their Facebook page”
• “When I go to the heart doctor, my interpreter likes to talk to the doctor in English. He
tells everyone that he went to medical school back in his country. The doctor hardly
pays any attention to me, and the interpreter tells me not to ask so many questions,
just to do what the doctor instructs me to do.”
14
Healthcare Interpreter Code of Ethics
(Washington state DSHS requires the following)
• Accuracy
• Cultural sensitivity-Courtesy
• Confidentiality
• Proficiency
• Compensation
• Nondiscrimination
• Self-representation
• Impartiality-Conflict of Interest
• Professional demeanor
• Reporting obstacles to practice
• Professional development
• Scope of Practice
15
Accuracy and Completeness
 Transmit the meaning.
 Do not leave anything out.
 Do not add anything unless you claim it.
 Do not change anything.
 Transmit the tone. This is vital information.
 Transmit any meaning from body language.
 Clarify your own understanding of statements from all parties.
 Notice and be aware of the integrity of the message. If you do
not understand it, maybe it was ambiguous or mistaken. 16
Accuracy & Ethical Problem-Solving
 Let the provider/patient/client know if you do not understand what they say
 Have them explain it. Do not guess.
 Write down numbers or names or meds so that you can repeat them correctly
 Ask provider/patient/client to repeat if you forgot something they said
 Convey speaker’s emotion, anger, bad words
 It is fine to remind all parties that you are transmitting the message, not originating it
 Correct any mistakes you make ASAP, openly
 Acknowledge when interpreter limitations may affect accuracy of interpretation
(lack of medical knowledge, inability to understand the dialect
or the speech of a patient, etc.)
17
Accuracy stories
 The male patient was very angry over treatment set-backs. In the office visit
he used rude, aggressive comments to the doctor to express how he felt about
his treatment.
 The female interpreter was shocked to hear the patient express such rude and
angry sentiments to the doctor, who was trying to do his best to take care of
the patient. She did not ever swear in English, and did not know how to
interpret the exact equivalent of the words. Being a very polite older woman,
she could not bring herself to yell at the doctor.
 The interpreter knew that the doctor needed to know that the patient was
saying rude and angry things, so she used her interpreter voice:
 “The interpreter wants the doctor to know that the patient is using rude and
angry words to express himself about the treatment plan.”
 Because the doctor now knew that the patient was very angry and had even
used rude words in front of the care team, the doctor could effectively
engage the patient in further discussion. 18
Accuracy: Scenario Quiz
 The provider uses a term that you do not understand while
explaining to a patient what might go wrong during treatment.
You:
A) Leave out that sentence, as the patient already has enough to worry about.
B) Say to the patient: “I did not understand part of what the doctor said about
risks of treatment. Sorry.”
C) Say to both the doctor and the patient: “The interpreter asks the doctor to
break down the term X for me, so that I can interpret accurately. I do not
understand the term X.
D) Remember the last time you interpreted for a patient having this treatment,
and repeat what the doctor said that time.
19
Accuracy: Scenario Quiz
 The provider uses a term that you do not understand while
explaining to a patient what might go wrong during treatment.
You:
A) Leave out that sentence, as the patient already has enough to worry about.
B) Say to the patient: “I did not understand part of what the doctor said about
risks of treatment. Sorry.”
C) Say to both the doctor and the patient: “The interpreter asks the doctor to
break down the term X for me, so that I can interpret accurately. I do not
understand the term X.
D) Remember the last time you interpreted for a patient having this treatment,
and repeat what the doctor said that time.
20
Cultural Sensitivity and Respect
Interpreters cannot represent the patient/client culturally,
but they can educate the staff on terms or cultural nuances
that would be generally considered rude in the patient’s or
client’s home country, so the staff can avoid unintentionally
insulting or even intimidating the patient
Staff should NOT ask interpreter to explain
the client’s or patient’s needs or behavior.
21
Cultural Sensitivity--Story
An elderly woman is diagnosed with cancer. Her family does not allow care
team to disclose diagnosis to her, so she is aware only that she has to undergo
treatment. As her cancer progresses and she is admitted into the hospital, a
new doctor comes into her room and talks about the fact that she has terminal
cancer.
The patient is truly surprised by the diagnosis, as well as by the fact that she
only has another couple of weeks to live. She is furious with her family because
they were not honest with her, and did not give her time to put her affairs in
order. She clearly states that her strong cultural value for honesty and full
sharing of information has been violated by all concerned.
The hospital where this happened now asks each patient, with no family
present and with the support of an interpreter, what information the patient
wants to receive. This direction from the patient is documented and honored.
22
Interpreter must be alert to cultural elements
 Own cultural identity: gender, age, education, economic status, race,
ethnicity, religion, clan, personal experiential formative factors (refugee,
single-mom, persecuted minority, disabled)
 Client/Patient cultural identity (stated and demonstrated): Deep values, desired
outcomes, fears, health beliefs, pattern of interacting with health system, comfort
requirements
 Cultural identity of various care staff attending patient: Constant comments to
patient about what he should do, what he should be glad or worried about,
externalization of their cultural beliefs onto the patient. (These can be painful to
have to interpret)
 American medical culture (biomedical): High value for directness, speed of
decision-making, commitment to course of action
 Research culture: The conceptual underpinning of research is that it helps
people in the future, not the patient necessarily. This is a difficult concept
to represent and to interpret 23
Cultural Sensitivity--Scenario Quiz
 At an ER visit for a child with bronchitis, the ER team is concerned by bruises
and raised welts on the child’s back. The ER team suspects child abuse.
 The SE Asian interpreter, who suspects that the bruises on the child are
caused by the well-accepted and commonly used practices of coining and
cupping, appropriately does the following:
A) Mentions aside to the doctor, without including the parents, that these are
ignorant peasants who employ superstitious home remedies, and that the
bruises are evidence that this family is going to be hard to convince to use
modern antibiotics.
B) Suggests to the family, transparently letting the team know what he is doing,
that the family describe the origin of the marks on the child’s back to the team.
C) The interpreter gives a lecture in English to the care team about how important
and valid home medical remedies are. The interpreter then says to the family
in their language that he has defended their practice of cupping and coining
and has saved them from being arrested for child-abuse. 24
Cultural Sensitivity--Scenario Quiz
 At an ER visit for a child with bronchitis, the ER team is concerned by bruises
and raised welts on the child’s back. The ER team suspects child abuse.
 The SE Asian interpreter, who suspects that the bruises on the child are
caused by the well-accepted and commonly used practices of coining and
cupping, appropriately does the following:
A) Mentions aside to the doctor, without including the parents, that these are
ignorant peasants who employ superstitious home remedies, and that the
bruises are evidence that this family is going to be hard to convince to use
modern antibiotics.
B) Suggests to the family, transparently letting the team know what he is doing,
that the family describe the origin of the marks on the child’s back to the team.
C) The interpreter gives a lecture in English to the care team about how important
and valid home medical remedies are. The interpreter then says to the family
in their language that he has defended their practice of cupping and coining
and has saved them from being arrested for child-abuse. 25
Confidentiality
Protection of PHI—Protected Health Information
 A patient or client can be killed or ostracized if PHI is revealed to family or
members of their community
 Fear and distrust of interpreters is based on breaches of confidentiality known
in community
 Share PHI only within the health care (HC) organization
 Share PHI only for need-to-know reasons within HC
 Know the rules for what info can be left on a message machine or with family
member (if your role includes phone reminders)
 no diagnosis info
 no service info
 Personal electronic or written schedules should have NO PHI on them. Only use
the job reference number on your schedules. If it is absolutely necessary to
have client information: de-identify by keeping only first 5 letters of last name,
no first name, no DOB, no Med Rec number
 Dispose of written materials properly, including your own notes and voucher
copies and daily schedule:
 Ask the medical provider or staff to dispose of materials in a secure manner 26
Confidentiality: Regaining Public Trust
 In every continuing education class on ethics the interpreters tell stories of true
situations in which interpreters betrayed the confidentiality of their patients or clients
in extreme ways
 Interpreters have told the family members of patients and clients what had happened in
the private appointments
 Interpreters have told the community about pregnancies, abortions, birth control
choices, mental health conditions, cancer, HIV, autism, and many other specific
concerns, all learned under what should have been protected conditions of privacy
 Even small betrayals of confidentiality are forbidden. When meeting a patient or client
in the grocery store, an interpreter should only nod politely to the patient, and not
address her/him by name. Other people in the store might know that the interpreter
works in health or social services field, and might therefore jump to the conclusion that
the individual had met the interpreter while seeking care/services
27
Confidentiality: Ethical Actions
 Use clear messaging (scripts) to avoid sharing ANY info with family,
friends, coworkers who ask you about what is going on
 “If you are interested in the affairs of so-and-so, please ask him directly.”
 Report any breach of confidentiality (this includes losing documents with
private information) to the organization where the interpreter had the
assignment and/or to the agency which provided the interpreter with the
assignment (follow your contractual agreement protocols)
 Assure each patient or client that you are committed to protecting their privacy
 “As your interpreter, I do not mention or discuss anything about your care
to anyone.”
 Offer to withdraw from cases where patient or client might resent your knowing
their business
 “Because we both serve on the school board together, would you feel
more comfortable with a different interpreter?” 28
Confidentiality: Scenario Quiz
Interpreter returns to surgery waiting room while nurses clean up the
patient after some post-surgical vomiting. The mother-in-law of the patient
is waiting in the waiting room and engages the interpreter in conversation,
beginning with telling her what a great interpreter she is. The lady then
asks the interpreter if the patient has shared with her that the patient’s
unborn baby’s dates do not match her date of marriage.
The interpreter should:
A) Jump up and tell the mother-in-law that she should not gossip about
her pregnant daughter-in-law.
B) Say that information about the patient is not something that can be
discussed by members of the care team, including the interpreter.
C) Say that indeed there have been cases where babies have suspicious
dates, but surely her daughter-in-law is virtuous!?
29
Confidentiality: Scenario Quiz
Interpreter returns to surgery waiting room while nurses clean up the
patient after some post-surgical vomiting. The mother-in-law of the patient
is waiting in the waiting room and engages the interpreter in conversation,
beginning with telling her what a great interpreter she is. The lady then
asks the interpreter if the patient has shared with her that the patient’s
unborn baby’s dates do not match her date of marriage.
The interpreter should:
A) Jump up and tell the mother-in-law that she should not gossip about
her pregnant daughter-in-law.
B) Say that information about the patient is not something that can be
discussed by members of the care team, including the interpreter.
C) Say that indeed there have been cases where babies have suspicious
dates, but surely her daughter-in-law is virtuous!?
30
Competency vs. Proficiency
Why BOTH are important for best quality
31
Pair & Share Exercise
Turn to your neighbor and discuss how competence is different from
proficiency
32
Competency
Competency: as defined by Merriam-Webster,
is the ability to do something successfully or efficiently.
For interpreters, it means having the skills set of languages,
terminology, knowledge of culture and strong listening and memory
skills combined to function capably as an interpreter.
I want you to aspire to being MORE than merely a competent interpreter.
How does competent differ from being proficient?
33
Proficiency
Proficiency: as defined in the goals and targets for teachers of
world languages means one can--
Use language spontaneously, in unexpected situations,
to communicate any and all needs during that particular
moment in time
As interpreters, our profession is filled with daily unexpected,
spontaneous moments.
We often have to draw upon knowledge of history, geography,
world events, regional conflicts as well as traditional beliefs
and practices, in order to do a good job for our patients and
have their providers understand their needs and complaints. 34
Multiple Proficiencies!
Elements of proficiency required and expected of an interpreter
by patients, clients, DSHS staff, care team, purchasers of
service, regulatory agencies, community include:
 Linguistic proficiency in both languages
 medical and social service vocabulary
 terminologies related to science, math, biology, anatomy,
physiology, and human development
 Professional business management skills
 dependability, trustworthiness, customer service, time management
 Understanding of basic health care and social services operations
 Where to meet clients
 Check-in/check-out procedures
 Interpreter mechanics--what does the interpreter do?
 Interpreter ethics—what are the do’s and don’ts?
35
Multiple Proficiencies and Competencies!
20
7.5
7.5
5
2.55
2.5
5
5
5
5
5
2.5
5
2.5 2.5
The Wheel of Competencies
for Healthcare Interpreters
Language
Culture
Terminology
Mechanics
Language Access Law
Human Physiology
HIPPA
Professional Ethics
Professional Standards
Certification
Continuing Education
Self-Assessment
Hospital/Clinic Rules
Customer Service
Entrepreneurship
Membership in Prof Orgs
36What additional elements would you add to this list?
Which wheel do you want to ride on?
37
Your professional career will roll along
smoothly if you nurture ALL of your skills!
38
Your professional career’s future
Proficiency:
True Stories of Lack of Proficiency
 An interpreter signed up with an agency claiming to speak fluent Mandarin and
Cantonese. He did not speak Cantonese at all. The agency kept sending him to
Cantonese assignments, where he usually got someone in the patient’s family to
interpret for the patient while he stood there doing nothing (and getting paid.)
 An interpreter had a doctorate in the target language, English, but had such
poor pronunciation that the providers couldn’t understand him at all. Staff used
the family for communication.
 An interpreter accepted her very first assignment, which was at a specialty
hospital for a patient dying of leukemia. She did not have adequate knowledge
of medical terminology in either language. She was completely unorganized
and dressed inappropriately for the encounter holding a cup of overflowing
coffee in her hand, which she proceeded to spill on several people and the
furniture. She was banned from ever coming back.
 An interpreter had no idea how to interpret this: “Your child’s head
circumference is at the 37th percentile for American children.” 39
Proficiency--Quiz
A brand new interpreter accepted an assignment from his agency to go to a
large psychiatric hospital for an encounter with a patient on a locked ward.
This interpreter began to realize that he was in over his head when the staff
on the locked ward told him that they wanted to brief him on safety
concerns for his patient before they locked him into the room with the
doctor.
The interpreter at that point should:
A) Disclose to the staff and to the doctor that he did not have any experience in
mental health and inpatient psych and was uncomfortable interpreting in
this environment.
B) If the doctor decides to continue the appointment, work very closely with
the doctor throughout the encounter to make sure that he understood the
meaning in both directions.
C) Go home afterward and start studying very hard to bring his proficiency in
psychiatric topics up to speed.
D) Tell his agency to send him to less specialized appointments while he
improves his knowledge bit by bit.
E) All of the above.
40
Proficiency--Quiz
A brand new interpreter accepted an assignment from his agency to go to a
large psychiatric hospital for an encounter with a patient on a locked ward.
This interpreter began to realize that he was in over his head when the staff
on the locked ward told him that they wanted to brief him on safety
concerns for his patient before they locked him into the room with the
doctor.
The interpreter at that point should:
A) Disclose to the staff and to the doctor that he did not have any experience in
mental health and inpatient psych and was uncomfortable interpreting in
this environment.
B) If the doctor decides to continue the appointment, work very closely with
the doctor throughout the encounter to make sure that he understood the
meaning in both directions.
C) Go home afterward and start studying very hard to bring his proficiency in
psychiatric topics up to speed.
D) Tell his agency to send him to less specialized appointments while he
improves his knowledge bit by bit.
E) ALL of the above!
41
Financial Gain
 Fee paid is the absolute limit of reimbursement.
 No additional payment can be requested, hinted at, or accepted,
from any party, by the interpreter
 Do not accept cash or gifts (even flowers)
 Return anything given to you via purchaser of services or agency
 Donate flowers to patients
 Share food or treats with dept.
 Never develop a financial conflict of interest
 Interpreting practice should have zero connection to any money-making
situation
 Never solicit business from patients or providers
 The provider requests an interpreter based on availability,
not for continuity or to favor a person
42
Financial Gain--Bad True Stories
 Interpreter solicited for translation jobs while interpreting for the client.
 Interpreter presented a gift to the parents of a newborn. The parents
then felt obligated to buy the interpreter something in return.
 The interpreter referred his brother-in-law to the patient to work on his
roof. The deal went sour, and the patient no longer felt comfortable
going to the clinic because he might run into the interpreter.
 The interpreter mentioned to all the clients that she runs a little
weekend store with ethnic food items in the neighborhood where
many of the clients live. 43
Financial Gain—Ethical Actions
 Remind patients and clients that they have the right to an
interpreter for free
 Remind patients and clients that your services are being
properly compensated
 Use a polite script to refuse gifts
 “It is my pleasure provide interpreter services to you. As professional
interpreters, we do not accept gifts from patients or clients.”
 Do not use the interpreting environment to get information with
which to make money, such as selling life insurance, babysitting,
or home IT services
 Do not refer patients and clients to any of your friends, family, or
acquaintances for services, even if they have the item or service
needed by the patient or client 44
Nondiscrimination - Personal Beliefs
 The interpreter must not treat a patient or client differently from
any other due to their likes or dislikes about the individual, their
beliefs, or their community
 During an encounter, an interpreter must keep their own beliefs
about what is right, good, or proper to themselves. The interpreter
must not inject his views into the session in any way. This refers to
what the staff say as well as to what the patient/client and family say
45
Non-Discrimination:
Bad True Stories
 Interpreter refused to see a patient because the patient had
tuberculosis/HIV positive/Hansen’s disease
 Interpreter refused to accept assignment with a client who is
transgender, because it made them feel uncomfortable
 Seattle AIDS Council complained that they have a tough time
finding interpreters in various common language groups to
interpret for their HIV-positive patients
 The interpreter refusing to interpret for a client that is from
a different tribe or clan
46
Nondiscrimination: Ethical Actions
 Truly refrain from showing your opinion about the client’s
practices or choices regarding their health care or social service
needs.
 No excuses,
 No slogans,
 No body language.
 The interpreter can do enormous damage to an individual by
showing their opinion about these topics.
 Save your opinions for arenas outside of interpreting encounters.
47
Non-Discrimination
 Nationally protected classes:
 gender, disability, race, color, national origin, age, socio-economic or
educational or marital status, religious or political beliefs, or sexual
orientation
 Other characteristics that interpreters must be careful to
avoid discriminating against:
 people from ethnic groups or affiliations that have been in opposition
to the interpreter’s ethnic group or affiliation
 people who have been or are presently incarcerated
 people with certain medical or mental afflictions
 people who have made certain life decisions that the interpreter
does not agree with. 48
Non-Discrimination:
Scenario Quiz
 Interpreter arrives at assignment and finds that the client is an elder
from a part of the community which has very strong religious beliefs that
are opposite to the interpreter’s personal beliefs. The interpreter’s first
impulse is to leave the encounter and not ask to be paid.
The professional interpreter should:
A) Greet the patient just as he would any other patient
B) Avoid the opportunity for personal discussion between himself and the patient
C) Let the receptionist know that he is not one of the religious affiliates of the
patient, that he himself has modern views
D) Interpret for the patient, paying great care to listen carefully to the meaning
from the patient and from the provider and to transmit it faithfully
E) All but C 49
Non-Discrimination:
Scenario Quiz
 Interpreter arrives at assignment and finds that the client is an elder
from a part of the community which has very strong religious beliefs that
are opposite to the interpreter’s personal beliefs. The interpreter’s first
impulse is to leave the encounter and not ask to be paid.
The professional interpreter should:
A) Greet the patient just as he would any other patient
B) Avoid the opportunity for personal discussion between himself and the patient
C) Let the receptionist know that he is not one of the religious affiliates of the
patient, that he himself has modern views
D) Interpret for the patient, paying great care to listen carefully to the meaning
from the patient and from the provider and to transmit it faithfully
E) All but C 50
Self-Representation
 Accurately state own certifications, training, and experience
 Certificate  Certification
 Ethical: Correct any mistaken idea that patients or staff have
of interpreter’s qualifications
 Examples of improper self-marketing:
 An interpreter referring to themselves as translators
 Taking assignments for languages without proper
certification/authorization
 Providing services that you are not qualified for
 Patient Navigator
 Insurance Specialist
51
Self-Representation: Quiz
 The interpreter has a reputation in the community as being very wise
about health care matters because of working at a major hospital as a
staff interpreter for many years. Many call him “doctor” out of respect,
as he is willing to listen to their health complaints and give them advice
on when to go to the emergency room
 The interpreter arrives at the bedside of a patient and recognizes a member
of his community who calls him “doctor.” The patient greets him this way in
the hospital room. The interpreter:
A) Explains to the patient right away that he is not a medical doctor, but that is his nickname
B) He accepts the title with a smile, and tells the real doctor that the community has
nicknamed him “doctor”, because he helps them out when they need help
C) Both A) and B)
52
Self-Representation: Quiz
 The interpreter has a reputation in the community as being very wise
about health care matters because of working at a major hospital as a
staff interpreter for many years. Many call him “doctor” out of respect,
as he is willing to listen to their health complaints and give them advice
on when to go to the emergency room
 The interpreter arrives at the bedside of a patient and recognizes a member
of his community who calls him “doctor.” The patient greets him this way in
the hospital room. The interpreter:
A) Explains to the patient right away that he is not a medical doctor, but that is his nickname
B) He accepts the title with a smile, and tells the real doctor that the community has
nicknamed him “doctor”, because he helps them out when they need help
C) Both A) and B)
53
Impartiality: Conflict of Interest
 The interpreter should not interpret for an individual if the individual
would feel embarrassed or constrained by the interpreter’s knowing
about their situation
 Patient’s disclosure of symptoms, needs, habits might be reduced or prevented
 The interpreter should not interpret for anyone if their own strong feelings
about the patient or client might get in the way of attentive, accurate,
respectful interpretation
 This does not mean you are free to decline assignments according to your likes
and dislikes, personal preferences. If interpreting for pregnancy terminations,
end-of-life, family planning, sexual assault or mental health are too
“uncomfortable” for you, or go against your personal beliefs, you should
choose a different area for your practice as a professional interpreter!
54
Impartiality: Quiz
 The interpreter has many relatives who are not accustomed yet to the U.S.
system of health care. These relatives need a great deal of encouragement to
disclose their symptoms and a great deal of explaining to understand their
condition and what they must do for self-care. The interpreter is not the
intimate caretaker for these people however, and there are plenty of politics
within the family. When another family member insists on interpreting, the
professional interpreter should:
A) Go with them to their appointments as a family member only
B) Tell them he will go with them the first few times but after that they will need to start
accepting other interpreters, knowing that they will never accept anyone else
C) Make an excuse every time so that he does not have to accompany his family members,
leading them to think that he is shirking his family duty
D) Educate family that professional interpreters are completely discreet and that
the clinic will provide an interpreter to them for free
E) A and D 55
Impartiality: Quiz
 The interpreter has many relatives who are not accustomed yet to the U.S.
system of health care. These relatives need a great deal of encouragement to
disclose their symptoms and a great deal of explaining to understand their
condition and what they must do for self-care. The interpreter is not the
intimate caretaker for these people however, and there are plenty of politics
within the family. When another family member insists on interpreting, the
professional interpreter should:
A) Go with them to their appointments as a family member only
B) Tell them he will go with them the first few times but after that they will need to start
accepting other interpreters, knowing that they will never accept anyone else
C) Make an excuse every time so that he does not have to accompany his family members,
leading them to think that he is shirking his family duty
D) Educate family that professional interpreters are completely discreet and that
the clinic will provide an interpreter to them for free
E) A and D 56
Professional Demeanor
 The interpreter’s function is to assist with communication
between parties in a health care or social services
environment.
 The interpreter supports this relationship best by keeping all
attention on the client’s/patient’s experience, and by being:
 Punctual and organized
 Prepared for the material and the environment of the encounter
 Dressed safely and respectfully for the setting
 Not distracting or calling attention to self by personal activity,
comments or needs
 Adhering to the code of ethics 57
Professional Demeanor: Quiz
 Common complaints about medical interpreters
made by patients or clients:
 A) Interpreter was on time
 B) Interpreter was cold and rude, acted superior or seemed
uninterested in the patient
 C) Interpreter was inappropriately dressed
 D) Interpreter was on the phone before, and, during encounter
 E) Interpreter was inappropriate in conversation while waiting
for the doctor
 F) All but A
58
Professional Demeanor: Quiz
 Common complaints about medical interpreters
made by patients or clients:
 A) Interpreter was on time
 B) Interpreter was cold and rude, acted superior or seemed
uninterested in the patient
 C) Interpreter was inappropriately dressed
 D) Interpreter was on the phone before, and, during encounter
 E) Interpreter was inappropriate in conversation while waiting
for the doctor
 F) All but A
59
Professional Demeanor: Quiz
 Common complaints about medical interpreters by medical
providers & staff:
A. Interpreter had to be prodded to interpret, lacked focus
B. Interpreter wandered away and took phone calls inappropriately
C. Interpreter was very good at working with shy clients
D. Interpreter was having side conversations with patient/family
E. Interpreter did not inspire confidence because of frequent hesitations
F. Interpreter engaged care team to talk about own health problems
G. All but C
60
Professional Demeanor: Quiz
 Common complaints about medical interpreters by medical
providers & staff:
A. Interpreter had to be prodded to interpret, lacked focus
B. Interpreter wandered away and took phone calls inappropriately
C. Interpreter was very good at working with shy clients
D. Interpreter was having side conversations with patient/family
E. Interpreter did not inspire confidence because of frequent hesitations
F. Interpreter engaged care team to talk about own health problems
G. All but C
61
Scope of Practice
 Interpreters/translators must not EVER:
 Counsel, refer, give advice, or express personal opinions to the
individuals for whom they are interpreting/translating;
 Engage in activities with clients that are not directly related to
providing interpreting
 Have unsupervised access to clients, including but not limited to
phoning clients directly, other than at the request of a provider or
staff member
 Market their services to clients while on the job, including but not
limited to, arranging services or appointments for clients in order
to create business for themselves; or
 Transport clients for any business, including social service,
L&I or medical appointments 62
Scope of Practice:
 Avoid Over-Familiarity with Patients and Family Members
 Do not give out your contact information
 Share only general and non-controversial information with
patients/clients
 Avoid making personal comments unless positive and non-intimate
 Do not encourage chatter about relationships, troubles, opinions
 Avoid Acting out of Role
 Do not comment on the patient’s health care situation AT ALL
 Do not make ANY specific recommendation to the patient/client
regarding his healthcare or services
 Fold any request for opinion or recommendation from the patient or
client back around into getting the patient/client to ask the staff 63
Scope of Practice: Ethical Actions
 Use a polite script to decline discussing personal subjects
 Deflect or distract the individual from disclosing info about his care
or services, or remove yourself from the vicinity politely
 Educate individuals about boundaries of the interpreter role in general
64
Script to politely decline discussing personal subjects
 I prefer to keep my personal life separate from my work,
so I can focus on doing a great job for you.
 Today I’m here for YOU, to help you communicate your concerns
and understand your provider’s recommendations fully.
 I’m here to help you communicate with your provider. We
should stay focused on you and your immediate concerns.
65
Scope of Practice-Quiz
 Interpreter sits on far side of waiting room. Patient’s wife
comes over to interpreter and says that she is worried
because her husband has had chest pain but will only be
telling the nurse that he has indigestion.
The interpreter should:
A) Say that chest pain is a bad sign, the patient probably has a heart
condition, and the wife should speak up to the nurse
B) Say that any concern from the patient or the family is important for
the care team to know about, and would the wife like the interpreter
to accompany her now to talk to the nurse?
C) Go over to the patient and tell him that he is playing with fire to
keep important symptoms hidden, and that he should go right now
to talk to the nurse, as every minute might be his last
D) Tell the wife that if it is the patient’s time to die, God will take
him, otherwise he will be fine
66
Scope of Practice-Quiz
 Interpreter sits on far side of waiting room. Patient’s wife
comes over to interpreter and says that she is worried
because her husband has had chest pain but will only be
telling the nurse that he has indigestion.
The interpreter should:
A) Say that chest pain is a bad sign, the patient probably has a heart
condition, and the wife should speak up to the nurse
B) Say that any concern from the patient or the family is important
for the care team to know about, and would the wife like the
interpreter to accompany her now to talk to the nurse?
C) Go over to the patient and tell him that he is playing with fire to
keep important symptoms hidden, and that he should go right now
to talk to the nurse, as every minute might be his last
D) Tell the wife that if it is the patient’s time to die, God will take
him, otherwise he will be fine
67
Reporting Obstacles to Practice:
Ethical Requirement
 Withdraw from encounter if you have:
 doubts about your own competence to manage the
 lack of knowledge,
 emotional involvement,
 impairment
 Unease about encounter violating code of ethics in some
 might make client or patient uncomfortable;
 client or patient is too close (family or friend) to interpreter,
thus an ethics violation
68
Reporting Obstacles: Quiz
 Interpreter arrives at encounter in the clinic to find that the patient
is a teenage girl who has been raped in an incest situation. As the
provider begins to solicit details, the interpreter feels herself getting
more and more tense, angry, upset and distracted, even fearful. Why?
She and her older sister experienced the same thing
 The interpreter should:
A) Realize that she will not be able to interpret properly for patient
B) Realize that it is critical to not add any pain or embarrassment to the patient
by stating that she cannot interpret because of the topic
C) Run out of the room and pretend that she has food poisoning
D) Quietly use the Interpreter Voice to say to both patient and provider that she
must excuse herself because she is not sure that she can do a good enough
job of interpreting, but she will stay until another method of language
support is arranged for. She says that what the patient and doctor will talk
about is very important, and she wants to make sure that they have the
services of an interpreter who can properly and professionally perform
E) All but one of the above
69
Reporting Obstacles: Quiz
 Interpreter arrives at encounter in the clinic to find that the patient
is a teenage girl who has been raped in an incest situation. As the
provider begins to solicit details, the interpreter feels herself getting
more and more tense, angry, upset and distracted, even fearful. Why?
She and her older sister experienced the same thing
 The interpreter should:
A) Realize that she will not be able to interpret properly for patient
B) Realize that it is critical to not add any pain or embarrassment to the patient
by stating that she cannot interpret because of the topic
C) Run out of the room and pretend that she has food poisoning
D) Quietly use the Interpreter Voice to say to both patient and provider that she
must excuse herself because she is not sure that she can do a good enough
job of interpreting, but she will stay until another method of language
support is arranged for. She says that what the patient and doctor will talk
about is very important, and she wants to make sure that they have the
services of an interpreter who can properly and professionally perform
E) All but C
70
Professional Development-Ethical Actions
 Assess your knowledge
 Language (both source and target languages)
 Technical
 Cultural
 Community
 Healthcare System
 Address your knowledge gaps via TRAINING/CONTINUING EDUCATION
 Assess your skills
 Memory
 Ability to maintain Register
 Managing the flow
 Use of diplomatic scripts
 Address your skills gaps
71
Professional Development-Ethical Actions
 Assess your knowledge (technical understanding, language,
culture, community, system), then
 Address your knowledge gaps, then
 Assess your skills, then
 Address your skill gaps
 Training; CE; professional relationships and consultation
72
Professional Development-Quiz
 The interpreter receives a steady stream of solicitations from
agencies to interpret. No one ever checks her specific knowledge of
encounter topics, assuming that she is interpreting accurately and
completely. She should:
A) Spend all of her time doing assignments, as that helps the most
people in the community.
B) Keep a log of every instance in which she had trouble understanding
the content of a conversation.
C) Stay away from interpreter training classes because that would let
everyone know that she had more to learn.
D) Target the areas in which she has identified deficits in her knowledge
or skill set, and take classes in those subjects.
E) Two of the above. Which two?
73
Professional Development-Quiz
 The interpreter receives a steady stream of solicitations from
agencies to interpret. No one ever checks her specific knowledge of
encounter topics, assuming that she is interpreting accurately and
completely. She should:
A) Spend all of her time doing assignments, as that helps the most
people in the community.
B) Keep a log of every instance in which she had trouble understanding
the content of a conversation.
C) Stay away from interpreter training classes because that would let
everyone know that she had more to learn.
D) Target the areas in which she has identified deficits in her knowledge
or skill set, and take classes in those subjects.
E) B and D
74
Review
 The Code of Ethics governs role boundaries and expectations
for behavior and performance of all interpreters:
 preparation for the work, initially and continuously
 accountability for a good product being delivered (completeness,
accuracy)
 cultural awareness and support of patient and client values and
needs
 protection of patients and clients from financial predation
 protection of interpreter and patient/client from inappropriate
mixing of social and professional interactions
 protection of patient/client from control by interpreters 75
Questions?
Contact Information for:
 Eliana Lobo
Lobo Language Access
InterpreterTrainer@outlook.com
 Jazmin Manjarrez
OSTI Vice President
classymexi55@gmail.com
76

More Related Content

What's hot

Ethics & juresprudence by Dr. Abhishek gaur (8741095005)
Ethics & juresprudence by Dr. Abhishek gaur (8741095005)Ethics & juresprudence by Dr. Abhishek gaur (8741095005)
Ethics & juresprudence by Dr. Abhishek gaur (8741095005)Dr. Abhishek Ashok Sharma
 
Ethics in orthodontia/ ethics in dentistry
Ethics in orthodontia/ ethics in dentistryEthics in orthodontia/ ethics in dentistry
Ethics in orthodontia/ ethics in dentistryAkhter Gul
 
Principles of ethics and code of professional conduct
Principles of ethics and code of professional conductPrinciples of ethics and code of professional conduct
Principles of ethics and code of professional conductOnessa Morante
 
Ciara Smith resume1
Ciara Smith resume1Ciara Smith resume1
Ciara Smith resume1ciara Smith
 
Laws and dentistry
Laws and dentistryLaws and dentistry
Laws and dentistryrineekhanna
 
Legal and ethical issues in critical care nursing
Legal and ethical issues in critical care nursingLegal and ethical issues in critical care nursing
Legal and ethical issues in critical care nursingNursing Path
 
Introduction to nursing for certified nursing assistants ch 1 4
Introduction to nursing for  certified nursing assistants ch 1 4Introduction to nursing for  certified nursing assistants ch 1 4
Introduction to nursing for certified nursing assistants ch 1 4MillieSefranek
 

What's hot (9)

Ethics & juresprudence by Dr. Abhishek gaur (8741095005)
Ethics & juresprudence by Dr. Abhishek gaur (8741095005)Ethics & juresprudence by Dr. Abhishek gaur (8741095005)
Ethics & juresprudence by Dr. Abhishek gaur (8741095005)
 
corner stone- CYW
corner stone- CYWcorner stone- CYW
corner stone- CYW
 
Ethics in orthodontia/ ethics in dentistry
Ethics in orthodontia/ ethics in dentistryEthics in orthodontia/ ethics in dentistry
Ethics in orthodontia/ ethics in dentistry
 
Professionalism in Medicine
Professionalism in MedicineProfessionalism in Medicine
Professionalism in Medicine
 
Principles of ethics and code of professional conduct
Principles of ethics and code of professional conductPrinciples of ethics and code of professional conduct
Principles of ethics and code of professional conduct
 
Ciara Smith resume1
Ciara Smith resume1Ciara Smith resume1
Ciara Smith resume1
 
Laws and dentistry
Laws and dentistryLaws and dentistry
Laws and dentistry
 
Legal and ethical issues in critical care nursing
Legal and ethical issues in critical care nursingLegal and ethical issues in critical care nursing
Legal and ethical issues in critical care nursing
 
Introduction to nursing for certified nursing assistants ch 1 4
Introduction to nursing for  certified nursing assistants ch 1 4Introduction to nursing for  certified nursing assistants ch 1 4
Introduction to nursing for certified nursing assistants ch 1 4
 

Similar to Healthcare interpreter ethics

Occupational Therapy Competency Standards
Occupational Therapy Competency StandardsOccupational Therapy Competency Standards
Occupational Therapy Competency StandardsStephan Van Breenen
 
Public Relationship In Healthcare
Public Relationship In HealthcarePublic Relationship In Healthcare
Public Relationship In HealthcareAvinash Advani
 
Online training
Online trainingOnline training
Online trainingHeather
 
Online training
Online training Online training
Online training Heather
 
ASVINI. M - CODE OF ETHICS ASS.pptx
ASVINI. M - CODE OF ETHICS ASS.pptxASVINI. M - CODE OF ETHICS ASS.pptx
ASVINI. M - CODE OF ETHICS ASS.pptxDeepikaLingam2
 
DIETETICNUTRITIONAL CODE OF ETHICS1. The dietetics practition
DIETETICNUTRITIONAL CODE OF ETHICS1. The dietetics practitionDIETETICNUTRITIONAL CODE OF ETHICS1. The dietetics practition
DIETETICNUTRITIONAL CODE OF ETHICS1. The dietetics practitionLinaCovington707
 
Roles of A Doctor.pptx
Roles of A Doctor.pptxRoles of A Doctor.pptx
Roles of A Doctor.pptxPradeep Pande
 
12 legal & ethical issues
12 legal & ethical issues 12 legal & ethical issues
12 legal & ethical issues NaveenJyothi
 
Overview of doctor’s roles and duties ghaiath
Overview of doctor’s roles and duties ghaiathOverview of doctor’s roles and duties ghaiath
Overview of doctor’s roles and duties ghaiathDr Ghaiath Hussein
 
Position Statement competency standards for the registered nurse.docx
Position Statement competency standards for the registered nurse.docxPosition Statement competency standards for the registered nurse.docx
Position Statement competency standards for the registered nurse.docxwrite5
 
Ethics and patient care standards
Ethics and patient care standardsEthics and patient care standards
Ethics and patient care standardsPARVATHY GOPI
 
Code of Ethics, Code of professional conduct, Legal aspects in Nursing .pptx
Code of Ethics, Code of professional conduct, Legal aspects in Nursing .pptxCode of Ethics, Code of professional conduct, Legal aspects in Nursing .pptx
Code of Ethics, Code of professional conduct, Legal aspects in Nursing .pptxRenjini R
 
Australian Healthcare System part 2
 Australian Healthcare System part 2 Australian Healthcare System part 2
Australian Healthcare System part 2Stephan Van Breenen
 

Similar to Healthcare interpreter ethics (20)

Interpreting in Health Care Progress Made & Challenges Ahead
Interpreting in Health Care Progress Made & Challenges Ahead Interpreting in Health Care Progress Made & Challenges Ahead
Interpreting in Health Care Progress Made & Challenges Ahead
 
Occupational Therapy Competency Standards
Occupational Therapy Competency StandardsOccupational Therapy Competency Standards
Occupational Therapy Competency Standards
 
Public Relationship In Healthcare
Public Relationship In HealthcarePublic Relationship In Healthcare
Public Relationship In Healthcare
 
2. ICN Code for Nursing Ethics.pdf
2. ICN Code for Nursing Ethics.pdf2. ICN Code for Nursing Ethics.pdf
2. ICN Code for Nursing Ethics.pdf
 
Online training
Online trainingOnline training
Online training
 
Online training
Online training Online training
Online training
 
ASVINI. M - CODE OF ETHICS ASS.pptx
ASVINI. M - CODE OF ETHICS ASS.pptxASVINI. M - CODE OF ETHICS ASS.pptx
ASVINI. M - CODE OF ETHICS ASS.pptx
 
Ethics in Counseling
Ethics in CounselingEthics in Counseling
Ethics in Counseling
 
DIETETICNUTRITIONAL CODE OF ETHICS1. The dietetics practition
DIETETICNUTRITIONAL CODE OF ETHICS1. The dietetics practitionDIETETICNUTRITIONAL CODE OF ETHICS1. The dietetics practition
DIETETICNUTRITIONAL CODE OF ETHICS1. The dietetics practition
 
unit I.pptx
unit I.pptxunit I.pptx
unit I.pptx
 
Roles of A Doctor.pptx
Roles of A Doctor.pptxRoles of A Doctor.pptx
Roles of A Doctor.pptx
 
12 legal & ethical issues
12 legal & ethical issues 12 legal & ethical issues
12 legal & ethical issues
 
Overview of doctor’s roles and duties ghaiath
Overview of doctor’s roles and duties ghaiathOverview of doctor’s roles and duties ghaiath
Overview of doctor’s roles and duties ghaiath
 
A Healthcare Interpreter's Best Friend
A Healthcare Interpreter's Best FriendA Healthcare Interpreter's Best Friend
A Healthcare Interpreter's Best Friend
 
Position Statement competency standards for the registered nurse.docx
Position Statement competency standards for the registered nurse.docxPosition Statement competency standards for the registered nurse.docx
Position Statement competency standards for the registered nurse.docx
 
Ethics and patient care standards
Ethics and patient care standardsEthics and patient care standards
Ethics and patient care standards
 
Code of Ethics, Code of professional conduct, Legal aspects in Nursing .pptx
Code of Ethics, Code of professional conduct, Legal aspects in Nursing .pptxCode of Ethics, Code of professional conduct, Legal aspects in Nursing .pptx
Code of Ethics, Code of professional conduct, Legal aspects in Nursing .pptx
 
Ethics and jurisprudence
Ethics and jurisprudenceEthics and jurisprudence
Ethics and jurisprudence
 
Australian Healthcare System part 2
 Australian Healthcare System part 2 Australian Healthcare System part 2
Australian Healthcare System part 2
 
Legal and Practical Implications of the Limited-English Speaking (LEP) Patient
Legal and Practical Implications of the  Limited-English Speaking (LEP) Patient Legal and Practical Implications of the  Limited-English Speaking (LEP) Patient
Legal and Practical Implications of the Limited-English Speaking (LEP) Patient
 

More from Eliana Lobo

Interpreting for Well Visits_ELobo.pptx
Interpreting for Well Visits_ELobo.pptxInterpreting for Well Visits_ELobo.pptx
Interpreting for Well Visits_ELobo.pptxEliana Lobo
 
What You Need to Succeed in VRI from Home - for NAETISL Jun 16 2022_ELobo.pptx
What You Need to Succeed in VRI from Home - for NAETISL Jun 16 2022_ELobo.pptxWhat You Need to Succeed in VRI from Home - for NAETISL Jun 16 2022_ELobo.pptx
What You Need to Succeed in VRI from Home - for NAETISL Jun 16 2022_ELobo.pptxEliana Lobo
 
Stress and Interpreting
Stress and Interpreting  Stress and Interpreting
Stress and Interpreting Eliana Lobo
 
How to Self Assess Your Skillset
How to Self Assess Your SkillsetHow to Self Assess Your Skillset
How to Self Assess Your SkillsetEliana Lobo
 
Working with Interpreters
Working with Interpreters  Working with Interpreters
Working with Interpreters Eliana Lobo
 
STRESS and Performance Anxiety During Test-Taking
STRESS and Performance Anxiety During Test-TakingSTRESS and Performance Anxiety During Test-Taking
STRESS and Performance Anxiety During Test-TakingEliana Lobo
 
Ethical conundrums in medical interpreting dshs-osti
Ethical conundrums in medical interpreting  dshs-ostiEthical conundrums in medical interpreting  dshs-osti
Ethical conundrums in medical interpreting dshs-ostiEliana Lobo
 
Push your performance
Push your performancePush your performance
Push your performanceEliana Lobo
 
Target Your Trainings - E. Lobo for NCIHC
Target Your Trainings - E. Lobo for NCIHCTarget Your Trainings - E. Lobo for NCIHC
Target Your Trainings - E. Lobo for NCIHCEliana Lobo
 

More from Eliana Lobo (9)

Interpreting for Well Visits_ELobo.pptx
Interpreting for Well Visits_ELobo.pptxInterpreting for Well Visits_ELobo.pptx
Interpreting for Well Visits_ELobo.pptx
 
What You Need to Succeed in VRI from Home - for NAETISL Jun 16 2022_ELobo.pptx
What You Need to Succeed in VRI from Home - for NAETISL Jun 16 2022_ELobo.pptxWhat You Need to Succeed in VRI from Home - for NAETISL Jun 16 2022_ELobo.pptx
What You Need to Succeed in VRI from Home - for NAETISL Jun 16 2022_ELobo.pptx
 
Stress and Interpreting
Stress and Interpreting  Stress and Interpreting
Stress and Interpreting
 
How to Self Assess Your Skillset
How to Self Assess Your SkillsetHow to Self Assess Your Skillset
How to Self Assess Your Skillset
 
Working with Interpreters
Working with Interpreters  Working with Interpreters
Working with Interpreters
 
STRESS and Performance Anxiety During Test-Taking
STRESS and Performance Anxiety During Test-TakingSTRESS and Performance Anxiety During Test-Taking
STRESS and Performance Anxiety During Test-Taking
 
Ethical conundrums in medical interpreting dshs-osti
Ethical conundrums in medical interpreting  dshs-ostiEthical conundrums in medical interpreting  dshs-osti
Ethical conundrums in medical interpreting dshs-osti
 
Push your performance
Push your performancePush your performance
Push your performance
 
Target Your Trainings - E. Lobo for NCIHC
Target Your Trainings - E. Lobo for NCIHCTarget Your Trainings - E. Lobo for NCIHC
Target Your Trainings - E. Lobo for NCIHC
 

Recently uploaded

Hierarchy of management that covers different levels of management
Hierarchy of management that covers different levels of managementHierarchy of management that covers different levels of management
Hierarchy of management that covers different levels of managementmkooblal
 
Biting mechanism of poisonous snakes.pdf
Biting mechanism of poisonous snakes.pdfBiting mechanism of poisonous snakes.pdf
Biting mechanism of poisonous snakes.pdfadityarao40181
 
Earth Day Presentation wow hello nice great
Earth Day Presentation wow hello nice greatEarth Day Presentation wow hello nice great
Earth Day Presentation wow hello nice greatYousafMalik24
 
Meghan Sutherland In Media Res Media Component
Meghan Sutherland In Media Res Media ComponentMeghan Sutherland In Media Res Media Component
Meghan Sutherland In Media Res Media ComponentInMediaRes1
 
History Class XII Ch. 3 Kinship, Caste and Class (1).pptx
History Class XII Ch. 3 Kinship, Caste and Class (1).pptxHistory Class XII Ch. 3 Kinship, Caste and Class (1).pptx
History Class XII Ch. 3 Kinship, Caste and Class (1).pptxsocialsciencegdgrohi
 
POINT- BIOCHEMISTRY SEM 2 ENZYMES UNIT 5.pptx
POINT- BIOCHEMISTRY SEM 2 ENZYMES UNIT 5.pptxPOINT- BIOCHEMISTRY SEM 2 ENZYMES UNIT 5.pptx
POINT- BIOCHEMISTRY SEM 2 ENZYMES UNIT 5.pptxSayali Powar
 
Types of Journalistic Writing Grade 8.pptx
Types of Journalistic Writing Grade 8.pptxTypes of Journalistic Writing Grade 8.pptx
Types of Journalistic Writing Grade 8.pptxEyham Joco
 
CELL CYCLE Division Science 8 quarter IV.pptx
CELL CYCLE Division Science 8 quarter IV.pptxCELL CYCLE Division Science 8 quarter IV.pptx
CELL CYCLE Division Science 8 quarter IV.pptxJiesonDelaCerna
 
ECONOMIC CONTEXT - PAPER 1 Q3: NEWSPAPERS.pptx
ECONOMIC CONTEXT - PAPER 1 Q3: NEWSPAPERS.pptxECONOMIC CONTEXT - PAPER 1 Q3: NEWSPAPERS.pptx
ECONOMIC CONTEXT - PAPER 1 Q3: NEWSPAPERS.pptxiammrhaywood
 
CARE OF CHILD IN INCUBATOR..........pptx
CARE OF CHILD IN INCUBATOR..........pptxCARE OF CHILD IN INCUBATOR..........pptx
CARE OF CHILD IN INCUBATOR..........pptxGaneshChakor2
 
Pharmacognosy Flower 3. Compositae 2023.pdf
Pharmacognosy Flower 3. Compositae 2023.pdfPharmacognosy Flower 3. Compositae 2023.pdf
Pharmacognosy Flower 3. Compositae 2023.pdfMahmoud M. Sallam
 
Solving Puzzles Benefits Everyone (English).pptx
Solving Puzzles Benefits Everyone (English).pptxSolving Puzzles Benefits Everyone (English).pptx
Solving Puzzles Benefits Everyone (English).pptxOH TEIK BIN
 
Interactive Powerpoint_How to Master effective communication
Interactive Powerpoint_How to Master effective communicationInteractive Powerpoint_How to Master effective communication
Interactive Powerpoint_How to Master effective communicationnomboosow
 
EPANDING THE CONTENT OF AN OUTLINE using notes.pptx
EPANDING THE CONTENT OF AN OUTLINE using notes.pptxEPANDING THE CONTENT OF AN OUTLINE using notes.pptx
EPANDING THE CONTENT OF AN OUTLINE using notes.pptxRaymartEstabillo3
 
Organic Name Reactions for the students and aspirants of Chemistry12th.pptx
Organic Name Reactions  for the students and aspirants of Chemistry12th.pptxOrganic Name Reactions  for the students and aspirants of Chemistry12th.pptx
Organic Name Reactions for the students and aspirants of Chemistry12th.pptxVS Mahajan Coaching Centre
 
call girls in Kamla Market (DELHI) 🔝 >༒9953330565🔝 genuine Escort Service 🔝✔️✔️
call girls in Kamla Market (DELHI) 🔝 >༒9953330565🔝 genuine Escort Service 🔝✔️✔️call girls in Kamla Market (DELHI) 🔝 >༒9953330565🔝 genuine Escort Service 🔝✔️✔️
call girls in Kamla Market (DELHI) 🔝 >༒9953330565🔝 genuine Escort Service 🔝✔️✔️9953056974 Low Rate Call Girls In Saket, Delhi NCR
 
Incoming and Outgoing Shipments in 1 STEP Using Odoo 17
Incoming and Outgoing Shipments in 1 STEP Using Odoo 17Incoming and Outgoing Shipments in 1 STEP Using Odoo 17
Incoming and Outgoing Shipments in 1 STEP Using Odoo 17Celine George
 
Full Stack Web Development Course for Beginners
Full Stack Web Development Course  for BeginnersFull Stack Web Development Course  for Beginners
Full Stack Web Development Course for BeginnersSabitha Banu
 
Crayon Activity Handout For the Crayon A
Crayon Activity Handout For the Crayon ACrayon Activity Handout For the Crayon A
Crayon Activity Handout For the Crayon AUnboundStockton
 

Recently uploaded (20)

Hierarchy of management that covers different levels of management
Hierarchy of management that covers different levels of managementHierarchy of management that covers different levels of management
Hierarchy of management that covers different levels of management
 
Biting mechanism of poisonous snakes.pdf
Biting mechanism of poisonous snakes.pdfBiting mechanism of poisonous snakes.pdf
Biting mechanism of poisonous snakes.pdf
 
Earth Day Presentation wow hello nice great
Earth Day Presentation wow hello nice greatEarth Day Presentation wow hello nice great
Earth Day Presentation wow hello nice great
 
Meghan Sutherland In Media Res Media Component
Meghan Sutherland In Media Res Media ComponentMeghan Sutherland In Media Res Media Component
Meghan Sutherland In Media Res Media Component
 
History Class XII Ch. 3 Kinship, Caste and Class (1).pptx
History Class XII Ch. 3 Kinship, Caste and Class (1).pptxHistory Class XII Ch. 3 Kinship, Caste and Class (1).pptx
History Class XII Ch. 3 Kinship, Caste and Class (1).pptx
 
POINT- BIOCHEMISTRY SEM 2 ENZYMES UNIT 5.pptx
POINT- BIOCHEMISTRY SEM 2 ENZYMES UNIT 5.pptxPOINT- BIOCHEMISTRY SEM 2 ENZYMES UNIT 5.pptx
POINT- BIOCHEMISTRY SEM 2 ENZYMES UNIT 5.pptx
 
Types of Journalistic Writing Grade 8.pptx
Types of Journalistic Writing Grade 8.pptxTypes of Journalistic Writing Grade 8.pptx
Types of Journalistic Writing Grade 8.pptx
 
CELL CYCLE Division Science 8 quarter IV.pptx
CELL CYCLE Division Science 8 quarter IV.pptxCELL CYCLE Division Science 8 quarter IV.pptx
CELL CYCLE Division Science 8 quarter IV.pptx
 
ECONOMIC CONTEXT - PAPER 1 Q3: NEWSPAPERS.pptx
ECONOMIC CONTEXT - PAPER 1 Q3: NEWSPAPERS.pptxECONOMIC CONTEXT - PAPER 1 Q3: NEWSPAPERS.pptx
ECONOMIC CONTEXT - PAPER 1 Q3: NEWSPAPERS.pptx
 
CARE OF CHILD IN INCUBATOR..........pptx
CARE OF CHILD IN INCUBATOR..........pptxCARE OF CHILD IN INCUBATOR..........pptx
CARE OF CHILD IN INCUBATOR..........pptx
 
Pharmacognosy Flower 3. Compositae 2023.pdf
Pharmacognosy Flower 3. Compositae 2023.pdfPharmacognosy Flower 3. Compositae 2023.pdf
Pharmacognosy Flower 3. Compositae 2023.pdf
 
Solving Puzzles Benefits Everyone (English).pptx
Solving Puzzles Benefits Everyone (English).pptxSolving Puzzles Benefits Everyone (English).pptx
Solving Puzzles Benefits Everyone (English).pptx
 
Interactive Powerpoint_How to Master effective communication
Interactive Powerpoint_How to Master effective communicationInteractive Powerpoint_How to Master effective communication
Interactive Powerpoint_How to Master effective communication
 
EPANDING THE CONTENT OF AN OUTLINE using notes.pptx
EPANDING THE CONTENT OF AN OUTLINE using notes.pptxEPANDING THE CONTENT OF AN OUTLINE using notes.pptx
EPANDING THE CONTENT OF AN OUTLINE using notes.pptx
 
OS-operating systems- ch04 (Threads) ...
OS-operating systems- ch04 (Threads) ...OS-operating systems- ch04 (Threads) ...
OS-operating systems- ch04 (Threads) ...
 
Organic Name Reactions for the students and aspirants of Chemistry12th.pptx
Organic Name Reactions  for the students and aspirants of Chemistry12th.pptxOrganic Name Reactions  for the students and aspirants of Chemistry12th.pptx
Organic Name Reactions for the students and aspirants of Chemistry12th.pptx
 
call girls in Kamla Market (DELHI) 🔝 >༒9953330565🔝 genuine Escort Service 🔝✔️✔️
call girls in Kamla Market (DELHI) 🔝 >༒9953330565🔝 genuine Escort Service 🔝✔️✔️call girls in Kamla Market (DELHI) 🔝 >༒9953330565🔝 genuine Escort Service 🔝✔️✔️
call girls in Kamla Market (DELHI) 🔝 >༒9953330565🔝 genuine Escort Service 🔝✔️✔️
 
Incoming and Outgoing Shipments in 1 STEP Using Odoo 17
Incoming and Outgoing Shipments in 1 STEP Using Odoo 17Incoming and Outgoing Shipments in 1 STEP Using Odoo 17
Incoming and Outgoing Shipments in 1 STEP Using Odoo 17
 
Full Stack Web Development Course for Beginners
Full Stack Web Development Course  for BeginnersFull Stack Web Development Course  for Beginners
Full Stack Web Development Course for Beginners
 
Crayon Activity Handout For the Crayon A
Crayon Activity Handout For the Crayon ACrayon Activity Handout For the Crayon A
Crayon Activity Handout For the Crayon A
 

Healthcare interpreter ethics

  • 1. Interpreter Ethics: Professional Role Boundaries for Healthcare Interpreters Eliana Lobo & Jazmin Manjarrez OSTI 2017
  • 2. Training objectives Be able to describe:  what a code of ethics is  why a code of ethics exists for every health profession  what the interpreter code of ethics covers Be able to explain what you would do in specific situations, such as:  what to do when you are asked to perform beyond your scope of practice  patient calls you at home to get your medical advice, and wants a ride to next appointment  care team asks you to report what patient said to you in waiting room  the family with a new baby tucks a gift in your briefcase  you made a mistake and you realize it a few minutes later  your sister runs a store with ethnic remedies that you think would work for the patient  you interpret for a person that you strongly disapprove of based on personal choices  you could make a lot of money interpreting for your extended family and close friends 2
  • 3. What is a code of ethics?  A code is a set of rules  Everyone within a profession or function is supposed to follow this set of rules  A code is usually set by the people who will live by it and whose reputation it defines  A code of ethics describes correct conduct in a certain role 3
  • 4. Health Care Code of Ethics (in general) Doctors, Nurses, Pharmacists, Therapists...  Protects patients from harm  correct info for good health outcomes  financial safety from predators  maintains privacy and empowers patient  Clarifies the role of the worker  scope, what does he do, what does he not do  professional boundaries and relationships  Defines a superior worker from those not so good  quality of work  professional habits 4
  • 5. Code of Ethics--Quiz A code of ethics provides guidelines for: A) rate of pay for workers B) role boundaries of the worker vis-a-vis clients C) how a worker can achieve a higher level within the hierarchy of his trade D) behavior of the worker towards his family 5
  • 6. National Codes of Ethics for Medical Interpreters  NCIHC http://www.ncihc.org/ethics-and-standards-of-practice  IMIA http://www.imiaweb.org/code/ 6
  • 7. NCIHC - Code of Ethics for Interpreters in Health Care 7 1. ‰The interpreter treats as confidential, within the treating team, all information learned in the performance of their professional duties, while observing relevant requirements regarding disclosure 2. ‰The interpreter strives to render the message accurately, conveying the content and spirit of the original message taking into consideration its cultural context 3. The interpreter maintains the boundaries of the professional role, refraining from personal involvement 4. ‰‰The interpreter continuously strives to develop awareness of his/her own and other (including biomedical) cultures encountered in the performance of their professional duties 5. ‰The interpreter strives to maintain impartiality & refrains from counseling, advising or projecting personal biases or beliefs 6. ‰‰When the patient’s health, well-being, or dignity is at risk, the interpreter may be justified in acting as an advocate. -Advocacy is understood as an action taken on behalf of an individual that goes beyond facilitating communication, with the intention of supporting good health outcomes. -Advocacy must only be undertaken after careful and thoughtful analysis of the situation and if other less intrusive actions have not resolved the problem 7. The interpreter treats all parties with respect 8. ‰The interpreter strives to continually further his/her knowledge and skills 9. ‰The interpreter must at all times act in a professional and ethical manner
  • 8. IMIA – Code of Ethics 8 The most important of these principles are: 1. Interpreters will maintain confidentiality of all assignment-related information 2. Interpreters will select the language and mode of interpretation that most accurately conveys the content and spirit of the messages of their clients 3. Interpreters will refrain from accepting assignments beyond their professional skills, language fluency, or level of training 4. Interpreters will refrain from accepting an assignment when family or close personal relationships affect impartiality 5. Interpreters will not interject personal opinions or counsel patients 6. Interpreters will not engage in interpretations that relate to issues outside the provision of health care services unless qualified to do so 7. Interpreters will engage in patient advocacy and in the intercultural mediation role of explaining cultural differences, or practices to health care providers /patients only when appropriate and necessary for communication purposes, using professional judgment 8. Interpreters will use skillful unobtrusive interventions so as not to interfere with the flow of communication in triadic medical settings 9. Interpreters will keep abreast of their evolving languages and medical terminology 10. Interpreters will participate in continuing education programs as available 11. Interpreters will seek to maintain ties with relevant professional organizations in order to be up-to-date with the latest professional standards and protocols 12. Interpreters will refrain from using their position to gain favors from clients.
  • 9. State Codes of Ethics for Medical Interpreters California – CHIA  http://c.ymcdn.com/sites/www.chiaonline.org/resource/resmgr/ docs/standards_chia.pdf Oregon - NCIHC  http://ostiweb.org/wp-content/uploads/2016/03/Interpreting- codes-of-ethics-that-apply-in-Oregon.pdf Washington - DSHS  https://www.dshs.wa.gov/fsa/language-testing-and-certification- program/code-ethics 9
  • 10. CA Endowment & CHIA –Ethical Principles 10 Each ethical principles is to be considered in the context of the health and well-being of the patient. 1. Confidentiality Interpreters treat all information learned during the interpreting as confidential. 2. Impartiality Interpreters are aware of the need to identify any potential or actual 10 Executive Summary conflicts of interest, as well as any personal judgments, values, beliefs or opinions that may lead to preferential behavior or bias affecting the quality and accuracy of the interpreting performance. 3. Respect for individuals and their communities Interpreters strive to support mutually respectful relationships between all three parties in the interaction (patient, provider and interpreter), while supporting the health and well-being of the patient as the highest priority of all healthcare professionals. 4. Professionalism and integrity Interpreters conduct themselves in a manner consistent with the professional standards and ethical principles of the healthcare interpreting profession. 5. Accuracy and completeness Interpreters transmit the content, spirit and cultural context of the original message into the target language, making it possible for patient and provider to communicate effectively. 5. Cultural responsiveness Interpreters seek to understand how diversity and cultural similarities and differences have a fundamental impact on the healthcare encounter. Interpreters play a critical role in identifying cultural issues and considering how and when to move to a cultural clarifier r ole. Developing cultural sensitivity and cultural responsiveness is a life-long process that begins with an introspective look at oneself.
  • 11. Oregon Code of Ethics for Healthcare Interpreters Code of Ethics and Standards of Practice for Interpreters in Health Care 1. Health care interpreters must adhere to the National Code of Ethics for Interpreters in Health Care as established by the National Council on Interpreting in Health Care. 2. Healthcare interpreters must adhere to the National Standards of Practice for Interpreters in Health Care as established by the National Council on Interpreting in Health Care. 11
  • 12. WA DSHS–Code of Ethics for Medical Interpreters 12 Accuracy. Interpreters/translators must always express the source language message in a thorough and faithful manner. They must: omit/add nothing; Give consideration to linguistic variations in both the source and target languages; Conserve the tone and spirit of the source language. Cultural sensitivity-courtesy. Interpreters/translators must be culturally sensitive, & respectful of whom they serve. Confidentiality. Interpreters/translators must not divulge any information publicly or privately obtained through their assignments, including, but not limited to, information gained through access to documents or other written materials. Proficiency. Interpreters/translators must meet the minimum proficiency standard set by DSHS. Compensation. Interpreters/translators must: Not accept additional money, consideration, or favors for services reimbursed by the department. The fee schedule agreed to between the contracted language services providers and the department shall be the maximum compensation accepted. Nondiscrimination. Interpreters/translators must: Always be impartial and unbiased; Not discriminate on the basis of gender, disability, race, color, national origin, age, socio-economic or educational or marital status, religious or political beliefs, or sexual orientation; and Refuse or withdraw from an assignment, without threat or retaliation, if they are unable to perform the required service in an ethical manner. Self-representation. Interpreters/translators must accurately and completely represent their certifications, training, and experience. Impartiality-conflict of interest. Interpreters/translators must disclose to the department any real or perceived conflicts of interest that would affect their professional objectivity. Professional demeanor. Interpreters/translators must be punctual, prepared, and dressed in a manner appropriate, and not distracting for the situation. Scope of practice. Interpreters/translators must not: Counsel, refer, give advice, or express personal opinions to the individuals for whom they are interpreting/translating;
  • 13. Pair & Share Exercise Turn to your neighbor and share an ethical quandary or consideration from your work experience, or from a personal observation 13
  • 14. Interpreter Code of Ethics Why did we fight so hard to create it? True patient stories: • “My interpreter told me that the other interpreter in town would gossip about me, so I should always get appointments that would fit with his schedule. This slowed down my course of treatment so much that I lost my insurance before I could finish treatment, and now I can’t afford to go to the doctor.” • “My interpreter was with me when I had to terminate my pregnancy due to very severe abnormalities in the fetus. The interpreter keeps telling me that God does not allow abortions. I am so depressed that I cannot take care of myself or my two children.” • “My interpreter took a picture of me and posted it on their Facebook page” • “When I go to the heart doctor, my interpreter likes to talk to the doctor in English. He tells everyone that he went to medical school back in his country. The doctor hardly pays any attention to me, and the interpreter tells me not to ask so many questions, just to do what the doctor instructs me to do.” 14
  • 15. Healthcare Interpreter Code of Ethics (Washington state DSHS requires the following) • Accuracy • Cultural sensitivity-Courtesy • Confidentiality • Proficiency • Compensation • Nondiscrimination • Self-representation • Impartiality-Conflict of Interest • Professional demeanor • Reporting obstacles to practice • Professional development • Scope of Practice 15
  • 16. Accuracy and Completeness  Transmit the meaning.  Do not leave anything out.  Do not add anything unless you claim it.  Do not change anything.  Transmit the tone. This is vital information.  Transmit any meaning from body language.  Clarify your own understanding of statements from all parties.  Notice and be aware of the integrity of the message. If you do not understand it, maybe it was ambiguous or mistaken. 16
  • 17. Accuracy & Ethical Problem-Solving  Let the provider/patient/client know if you do not understand what they say  Have them explain it. Do not guess.  Write down numbers or names or meds so that you can repeat them correctly  Ask provider/patient/client to repeat if you forgot something they said  Convey speaker’s emotion, anger, bad words  It is fine to remind all parties that you are transmitting the message, not originating it  Correct any mistakes you make ASAP, openly  Acknowledge when interpreter limitations may affect accuracy of interpretation (lack of medical knowledge, inability to understand the dialect or the speech of a patient, etc.) 17
  • 18. Accuracy stories  The male patient was very angry over treatment set-backs. In the office visit he used rude, aggressive comments to the doctor to express how he felt about his treatment.  The female interpreter was shocked to hear the patient express such rude and angry sentiments to the doctor, who was trying to do his best to take care of the patient. She did not ever swear in English, and did not know how to interpret the exact equivalent of the words. Being a very polite older woman, she could not bring herself to yell at the doctor.  The interpreter knew that the doctor needed to know that the patient was saying rude and angry things, so she used her interpreter voice:  “The interpreter wants the doctor to know that the patient is using rude and angry words to express himself about the treatment plan.”  Because the doctor now knew that the patient was very angry and had even used rude words in front of the care team, the doctor could effectively engage the patient in further discussion. 18
  • 19. Accuracy: Scenario Quiz  The provider uses a term that you do not understand while explaining to a patient what might go wrong during treatment. You: A) Leave out that sentence, as the patient already has enough to worry about. B) Say to the patient: “I did not understand part of what the doctor said about risks of treatment. Sorry.” C) Say to both the doctor and the patient: “The interpreter asks the doctor to break down the term X for me, so that I can interpret accurately. I do not understand the term X. D) Remember the last time you interpreted for a patient having this treatment, and repeat what the doctor said that time. 19
  • 20. Accuracy: Scenario Quiz  The provider uses a term that you do not understand while explaining to a patient what might go wrong during treatment. You: A) Leave out that sentence, as the patient already has enough to worry about. B) Say to the patient: “I did not understand part of what the doctor said about risks of treatment. Sorry.” C) Say to both the doctor and the patient: “The interpreter asks the doctor to break down the term X for me, so that I can interpret accurately. I do not understand the term X. D) Remember the last time you interpreted for a patient having this treatment, and repeat what the doctor said that time. 20
  • 21. Cultural Sensitivity and Respect Interpreters cannot represent the patient/client culturally, but they can educate the staff on terms or cultural nuances that would be generally considered rude in the patient’s or client’s home country, so the staff can avoid unintentionally insulting or even intimidating the patient Staff should NOT ask interpreter to explain the client’s or patient’s needs or behavior. 21
  • 22. Cultural Sensitivity--Story An elderly woman is diagnosed with cancer. Her family does not allow care team to disclose diagnosis to her, so she is aware only that she has to undergo treatment. As her cancer progresses and she is admitted into the hospital, a new doctor comes into her room and talks about the fact that she has terminal cancer. The patient is truly surprised by the diagnosis, as well as by the fact that she only has another couple of weeks to live. She is furious with her family because they were not honest with her, and did not give her time to put her affairs in order. She clearly states that her strong cultural value for honesty and full sharing of information has been violated by all concerned. The hospital where this happened now asks each patient, with no family present and with the support of an interpreter, what information the patient wants to receive. This direction from the patient is documented and honored. 22
  • 23. Interpreter must be alert to cultural elements  Own cultural identity: gender, age, education, economic status, race, ethnicity, religion, clan, personal experiential formative factors (refugee, single-mom, persecuted minority, disabled)  Client/Patient cultural identity (stated and demonstrated): Deep values, desired outcomes, fears, health beliefs, pattern of interacting with health system, comfort requirements  Cultural identity of various care staff attending patient: Constant comments to patient about what he should do, what he should be glad or worried about, externalization of their cultural beliefs onto the patient. (These can be painful to have to interpret)  American medical culture (biomedical): High value for directness, speed of decision-making, commitment to course of action  Research culture: The conceptual underpinning of research is that it helps people in the future, not the patient necessarily. This is a difficult concept to represent and to interpret 23
  • 24. Cultural Sensitivity--Scenario Quiz  At an ER visit for a child with bronchitis, the ER team is concerned by bruises and raised welts on the child’s back. The ER team suspects child abuse.  The SE Asian interpreter, who suspects that the bruises on the child are caused by the well-accepted and commonly used practices of coining and cupping, appropriately does the following: A) Mentions aside to the doctor, without including the parents, that these are ignorant peasants who employ superstitious home remedies, and that the bruises are evidence that this family is going to be hard to convince to use modern antibiotics. B) Suggests to the family, transparently letting the team know what he is doing, that the family describe the origin of the marks on the child’s back to the team. C) The interpreter gives a lecture in English to the care team about how important and valid home medical remedies are. The interpreter then says to the family in their language that he has defended their practice of cupping and coining and has saved them from being arrested for child-abuse. 24
  • 25. Cultural Sensitivity--Scenario Quiz  At an ER visit for a child with bronchitis, the ER team is concerned by bruises and raised welts on the child’s back. The ER team suspects child abuse.  The SE Asian interpreter, who suspects that the bruises on the child are caused by the well-accepted and commonly used practices of coining and cupping, appropriately does the following: A) Mentions aside to the doctor, without including the parents, that these are ignorant peasants who employ superstitious home remedies, and that the bruises are evidence that this family is going to be hard to convince to use modern antibiotics. B) Suggests to the family, transparently letting the team know what he is doing, that the family describe the origin of the marks on the child’s back to the team. C) The interpreter gives a lecture in English to the care team about how important and valid home medical remedies are. The interpreter then says to the family in their language that he has defended their practice of cupping and coining and has saved them from being arrested for child-abuse. 25
  • 26. Confidentiality Protection of PHI—Protected Health Information  A patient or client can be killed or ostracized if PHI is revealed to family or members of their community  Fear and distrust of interpreters is based on breaches of confidentiality known in community  Share PHI only within the health care (HC) organization  Share PHI only for need-to-know reasons within HC  Know the rules for what info can be left on a message machine or with family member (if your role includes phone reminders)  no diagnosis info  no service info  Personal electronic or written schedules should have NO PHI on them. Only use the job reference number on your schedules. If it is absolutely necessary to have client information: de-identify by keeping only first 5 letters of last name, no first name, no DOB, no Med Rec number  Dispose of written materials properly, including your own notes and voucher copies and daily schedule:  Ask the medical provider or staff to dispose of materials in a secure manner 26
  • 27. Confidentiality: Regaining Public Trust  In every continuing education class on ethics the interpreters tell stories of true situations in which interpreters betrayed the confidentiality of their patients or clients in extreme ways  Interpreters have told the family members of patients and clients what had happened in the private appointments  Interpreters have told the community about pregnancies, abortions, birth control choices, mental health conditions, cancer, HIV, autism, and many other specific concerns, all learned under what should have been protected conditions of privacy  Even small betrayals of confidentiality are forbidden. When meeting a patient or client in the grocery store, an interpreter should only nod politely to the patient, and not address her/him by name. Other people in the store might know that the interpreter works in health or social services field, and might therefore jump to the conclusion that the individual had met the interpreter while seeking care/services 27
  • 28. Confidentiality: Ethical Actions  Use clear messaging (scripts) to avoid sharing ANY info with family, friends, coworkers who ask you about what is going on  “If you are interested in the affairs of so-and-so, please ask him directly.”  Report any breach of confidentiality (this includes losing documents with private information) to the organization where the interpreter had the assignment and/or to the agency which provided the interpreter with the assignment (follow your contractual agreement protocols)  Assure each patient or client that you are committed to protecting their privacy  “As your interpreter, I do not mention or discuss anything about your care to anyone.”  Offer to withdraw from cases where patient or client might resent your knowing their business  “Because we both serve on the school board together, would you feel more comfortable with a different interpreter?” 28
  • 29. Confidentiality: Scenario Quiz Interpreter returns to surgery waiting room while nurses clean up the patient after some post-surgical vomiting. The mother-in-law of the patient is waiting in the waiting room and engages the interpreter in conversation, beginning with telling her what a great interpreter she is. The lady then asks the interpreter if the patient has shared with her that the patient’s unborn baby’s dates do not match her date of marriage. The interpreter should: A) Jump up and tell the mother-in-law that she should not gossip about her pregnant daughter-in-law. B) Say that information about the patient is not something that can be discussed by members of the care team, including the interpreter. C) Say that indeed there have been cases where babies have suspicious dates, but surely her daughter-in-law is virtuous!? 29
  • 30. Confidentiality: Scenario Quiz Interpreter returns to surgery waiting room while nurses clean up the patient after some post-surgical vomiting. The mother-in-law of the patient is waiting in the waiting room and engages the interpreter in conversation, beginning with telling her what a great interpreter she is. The lady then asks the interpreter if the patient has shared with her that the patient’s unborn baby’s dates do not match her date of marriage. The interpreter should: A) Jump up and tell the mother-in-law that she should not gossip about her pregnant daughter-in-law. B) Say that information about the patient is not something that can be discussed by members of the care team, including the interpreter. C) Say that indeed there have been cases where babies have suspicious dates, but surely her daughter-in-law is virtuous!? 30
  • 31. Competency vs. Proficiency Why BOTH are important for best quality 31
  • 32. Pair & Share Exercise Turn to your neighbor and discuss how competence is different from proficiency 32
  • 33. Competency Competency: as defined by Merriam-Webster, is the ability to do something successfully or efficiently. For interpreters, it means having the skills set of languages, terminology, knowledge of culture and strong listening and memory skills combined to function capably as an interpreter. I want you to aspire to being MORE than merely a competent interpreter. How does competent differ from being proficient? 33
  • 34. Proficiency Proficiency: as defined in the goals and targets for teachers of world languages means one can-- Use language spontaneously, in unexpected situations, to communicate any and all needs during that particular moment in time As interpreters, our profession is filled with daily unexpected, spontaneous moments. We often have to draw upon knowledge of history, geography, world events, regional conflicts as well as traditional beliefs and practices, in order to do a good job for our patients and have their providers understand their needs and complaints. 34
  • 35. Multiple Proficiencies! Elements of proficiency required and expected of an interpreter by patients, clients, DSHS staff, care team, purchasers of service, regulatory agencies, community include:  Linguistic proficiency in both languages  medical and social service vocabulary  terminologies related to science, math, biology, anatomy, physiology, and human development  Professional business management skills  dependability, trustworthiness, customer service, time management  Understanding of basic health care and social services operations  Where to meet clients  Check-in/check-out procedures  Interpreter mechanics--what does the interpreter do?  Interpreter ethics—what are the do’s and don’ts? 35
  • 36. Multiple Proficiencies and Competencies! 20 7.5 7.5 5 2.55 2.5 5 5 5 5 5 2.5 5 2.5 2.5 The Wheel of Competencies for Healthcare Interpreters Language Culture Terminology Mechanics Language Access Law Human Physiology HIPPA Professional Ethics Professional Standards Certification Continuing Education Self-Assessment Hospital/Clinic Rules Customer Service Entrepreneurship Membership in Prof Orgs 36What additional elements would you add to this list?
  • 37. Which wheel do you want to ride on? 37
  • 38. Your professional career will roll along smoothly if you nurture ALL of your skills! 38 Your professional career’s future
  • 39. Proficiency: True Stories of Lack of Proficiency  An interpreter signed up with an agency claiming to speak fluent Mandarin and Cantonese. He did not speak Cantonese at all. The agency kept sending him to Cantonese assignments, where he usually got someone in the patient’s family to interpret for the patient while he stood there doing nothing (and getting paid.)  An interpreter had a doctorate in the target language, English, but had such poor pronunciation that the providers couldn’t understand him at all. Staff used the family for communication.  An interpreter accepted her very first assignment, which was at a specialty hospital for a patient dying of leukemia. She did not have adequate knowledge of medical terminology in either language. She was completely unorganized and dressed inappropriately for the encounter holding a cup of overflowing coffee in her hand, which she proceeded to spill on several people and the furniture. She was banned from ever coming back.  An interpreter had no idea how to interpret this: “Your child’s head circumference is at the 37th percentile for American children.” 39
  • 40. Proficiency--Quiz A brand new interpreter accepted an assignment from his agency to go to a large psychiatric hospital for an encounter with a patient on a locked ward. This interpreter began to realize that he was in over his head when the staff on the locked ward told him that they wanted to brief him on safety concerns for his patient before they locked him into the room with the doctor. The interpreter at that point should: A) Disclose to the staff and to the doctor that he did not have any experience in mental health and inpatient psych and was uncomfortable interpreting in this environment. B) If the doctor decides to continue the appointment, work very closely with the doctor throughout the encounter to make sure that he understood the meaning in both directions. C) Go home afterward and start studying very hard to bring his proficiency in psychiatric topics up to speed. D) Tell his agency to send him to less specialized appointments while he improves his knowledge bit by bit. E) All of the above. 40
  • 41. Proficiency--Quiz A brand new interpreter accepted an assignment from his agency to go to a large psychiatric hospital for an encounter with a patient on a locked ward. This interpreter began to realize that he was in over his head when the staff on the locked ward told him that they wanted to brief him on safety concerns for his patient before they locked him into the room with the doctor. The interpreter at that point should: A) Disclose to the staff and to the doctor that he did not have any experience in mental health and inpatient psych and was uncomfortable interpreting in this environment. B) If the doctor decides to continue the appointment, work very closely with the doctor throughout the encounter to make sure that he understood the meaning in both directions. C) Go home afterward and start studying very hard to bring his proficiency in psychiatric topics up to speed. D) Tell his agency to send him to less specialized appointments while he improves his knowledge bit by bit. E) ALL of the above! 41
  • 42. Financial Gain  Fee paid is the absolute limit of reimbursement.  No additional payment can be requested, hinted at, or accepted, from any party, by the interpreter  Do not accept cash or gifts (even flowers)  Return anything given to you via purchaser of services or agency  Donate flowers to patients  Share food or treats with dept.  Never develop a financial conflict of interest  Interpreting practice should have zero connection to any money-making situation  Never solicit business from patients or providers  The provider requests an interpreter based on availability, not for continuity or to favor a person 42
  • 43. Financial Gain--Bad True Stories  Interpreter solicited for translation jobs while interpreting for the client.  Interpreter presented a gift to the parents of a newborn. The parents then felt obligated to buy the interpreter something in return.  The interpreter referred his brother-in-law to the patient to work on his roof. The deal went sour, and the patient no longer felt comfortable going to the clinic because he might run into the interpreter.  The interpreter mentioned to all the clients that she runs a little weekend store with ethnic food items in the neighborhood where many of the clients live. 43
  • 44. Financial Gain—Ethical Actions  Remind patients and clients that they have the right to an interpreter for free  Remind patients and clients that your services are being properly compensated  Use a polite script to refuse gifts  “It is my pleasure provide interpreter services to you. As professional interpreters, we do not accept gifts from patients or clients.”  Do not use the interpreting environment to get information with which to make money, such as selling life insurance, babysitting, or home IT services  Do not refer patients and clients to any of your friends, family, or acquaintances for services, even if they have the item or service needed by the patient or client 44
  • 45. Nondiscrimination - Personal Beliefs  The interpreter must not treat a patient or client differently from any other due to their likes or dislikes about the individual, their beliefs, or their community  During an encounter, an interpreter must keep their own beliefs about what is right, good, or proper to themselves. The interpreter must not inject his views into the session in any way. This refers to what the staff say as well as to what the patient/client and family say 45
  • 46. Non-Discrimination: Bad True Stories  Interpreter refused to see a patient because the patient had tuberculosis/HIV positive/Hansen’s disease  Interpreter refused to accept assignment with a client who is transgender, because it made them feel uncomfortable  Seattle AIDS Council complained that they have a tough time finding interpreters in various common language groups to interpret for their HIV-positive patients  The interpreter refusing to interpret for a client that is from a different tribe or clan 46
  • 47. Nondiscrimination: Ethical Actions  Truly refrain from showing your opinion about the client’s practices or choices regarding their health care or social service needs.  No excuses,  No slogans,  No body language.  The interpreter can do enormous damage to an individual by showing their opinion about these topics.  Save your opinions for arenas outside of interpreting encounters. 47
  • 48. Non-Discrimination  Nationally protected classes:  gender, disability, race, color, national origin, age, socio-economic or educational or marital status, religious or political beliefs, or sexual orientation  Other characteristics that interpreters must be careful to avoid discriminating against:  people from ethnic groups or affiliations that have been in opposition to the interpreter’s ethnic group or affiliation  people who have been or are presently incarcerated  people with certain medical or mental afflictions  people who have made certain life decisions that the interpreter does not agree with. 48
  • 49. Non-Discrimination: Scenario Quiz  Interpreter arrives at assignment and finds that the client is an elder from a part of the community which has very strong religious beliefs that are opposite to the interpreter’s personal beliefs. The interpreter’s first impulse is to leave the encounter and not ask to be paid. The professional interpreter should: A) Greet the patient just as he would any other patient B) Avoid the opportunity for personal discussion between himself and the patient C) Let the receptionist know that he is not one of the religious affiliates of the patient, that he himself has modern views D) Interpret for the patient, paying great care to listen carefully to the meaning from the patient and from the provider and to transmit it faithfully E) All but C 49
  • 50. Non-Discrimination: Scenario Quiz  Interpreter arrives at assignment and finds that the client is an elder from a part of the community which has very strong religious beliefs that are opposite to the interpreter’s personal beliefs. The interpreter’s first impulse is to leave the encounter and not ask to be paid. The professional interpreter should: A) Greet the patient just as he would any other patient B) Avoid the opportunity for personal discussion between himself and the patient C) Let the receptionist know that he is not one of the religious affiliates of the patient, that he himself has modern views D) Interpret for the patient, paying great care to listen carefully to the meaning from the patient and from the provider and to transmit it faithfully E) All but C 50
  • 51. Self-Representation  Accurately state own certifications, training, and experience  Certificate  Certification  Ethical: Correct any mistaken idea that patients or staff have of interpreter’s qualifications  Examples of improper self-marketing:  An interpreter referring to themselves as translators  Taking assignments for languages without proper certification/authorization  Providing services that you are not qualified for  Patient Navigator  Insurance Specialist 51
  • 52. Self-Representation: Quiz  The interpreter has a reputation in the community as being very wise about health care matters because of working at a major hospital as a staff interpreter for many years. Many call him “doctor” out of respect, as he is willing to listen to their health complaints and give them advice on when to go to the emergency room  The interpreter arrives at the bedside of a patient and recognizes a member of his community who calls him “doctor.” The patient greets him this way in the hospital room. The interpreter: A) Explains to the patient right away that he is not a medical doctor, but that is his nickname B) He accepts the title with a smile, and tells the real doctor that the community has nicknamed him “doctor”, because he helps them out when they need help C) Both A) and B) 52
  • 53. Self-Representation: Quiz  The interpreter has a reputation in the community as being very wise about health care matters because of working at a major hospital as a staff interpreter for many years. Many call him “doctor” out of respect, as he is willing to listen to their health complaints and give them advice on when to go to the emergency room  The interpreter arrives at the bedside of a patient and recognizes a member of his community who calls him “doctor.” The patient greets him this way in the hospital room. The interpreter: A) Explains to the patient right away that he is not a medical doctor, but that is his nickname B) He accepts the title with a smile, and tells the real doctor that the community has nicknamed him “doctor”, because he helps them out when they need help C) Both A) and B) 53
  • 54. Impartiality: Conflict of Interest  The interpreter should not interpret for an individual if the individual would feel embarrassed or constrained by the interpreter’s knowing about their situation  Patient’s disclosure of symptoms, needs, habits might be reduced or prevented  The interpreter should not interpret for anyone if their own strong feelings about the patient or client might get in the way of attentive, accurate, respectful interpretation  This does not mean you are free to decline assignments according to your likes and dislikes, personal preferences. If interpreting for pregnancy terminations, end-of-life, family planning, sexual assault or mental health are too “uncomfortable” for you, or go against your personal beliefs, you should choose a different area for your practice as a professional interpreter! 54
  • 55. Impartiality: Quiz  The interpreter has many relatives who are not accustomed yet to the U.S. system of health care. These relatives need a great deal of encouragement to disclose their symptoms and a great deal of explaining to understand their condition and what they must do for self-care. The interpreter is not the intimate caretaker for these people however, and there are plenty of politics within the family. When another family member insists on interpreting, the professional interpreter should: A) Go with them to their appointments as a family member only B) Tell them he will go with them the first few times but after that they will need to start accepting other interpreters, knowing that they will never accept anyone else C) Make an excuse every time so that he does not have to accompany his family members, leading them to think that he is shirking his family duty D) Educate family that professional interpreters are completely discreet and that the clinic will provide an interpreter to them for free E) A and D 55
  • 56. Impartiality: Quiz  The interpreter has many relatives who are not accustomed yet to the U.S. system of health care. These relatives need a great deal of encouragement to disclose their symptoms and a great deal of explaining to understand their condition and what they must do for self-care. The interpreter is not the intimate caretaker for these people however, and there are plenty of politics within the family. When another family member insists on interpreting, the professional interpreter should: A) Go with them to their appointments as a family member only B) Tell them he will go with them the first few times but after that they will need to start accepting other interpreters, knowing that they will never accept anyone else C) Make an excuse every time so that he does not have to accompany his family members, leading them to think that he is shirking his family duty D) Educate family that professional interpreters are completely discreet and that the clinic will provide an interpreter to them for free E) A and D 56
  • 57. Professional Demeanor  The interpreter’s function is to assist with communication between parties in a health care or social services environment.  The interpreter supports this relationship best by keeping all attention on the client’s/patient’s experience, and by being:  Punctual and organized  Prepared for the material and the environment of the encounter  Dressed safely and respectfully for the setting  Not distracting or calling attention to self by personal activity, comments or needs  Adhering to the code of ethics 57
  • 58. Professional Demeanor: Quiz  Common complaints about medical interpreters made by patients or clients:  A) Interpreter was on time  B) Interpreter was cold and rude, acted superior or seemed uninterested in the patient  C) Interpreter was inappropriately dressed  D) Interpreter was on the phone before, and, during encounter  E) Interpreter was inappropriate in conversation while waiting for the doctor  F) All but A 58
  • 59. Professional Demeanor: Quiz  Common complaints about medical interpreters made by patients or clients:  A) Interpreter was on time  B) Interpreter was cold and rude, acted superior or seemed uninterested in the patient  C) Interpreter was inappropriately dressed  D) Interpreter was on the phone before, and, during encounter  E) Interpreter was inappropriate in conversation while waiting for the doctor  F) All but A 59
  • 60. Professional Demeanor: Quiz  Common complaints about medical interpreters by medical providers & staff: A. Interpreter had to be prodded to interpret, lacked focus B. Interpreter wandered away and took phone calls inappropriately C. Interpreter was very good at working with shy clients D. Interpreter was having side conversations with patient/family E. Interpreter did not inspire confidence because of frequent hesitations F. Interpreter engaged care team to talk about own health problems G. All but C 60
  • 61. Professional Demeanor: Quiz  Common complaints about medical interpreters by medical providers & staff: A. Interpreter had to be prodded to interpret, lacked focus B. Interpreter wandered away and took phone calls inappropriately C. Interpreter was very good at working with shy clients D. Interpreter was having side conversations with patient/family E. Interpreter did not inspire confidence because of frequent hesitations F. Interpreter engaged care team to talk about own health problems G. All but C 61
  • 62. Scope of Practice  Interpreters/translators must not EVER:  Counsel, refer, give advice, or express personal opinions to the individuals for whom they are interpreting/translating;  Engage in activities with clients that are not directly related to providing interpreting  Have unsupervised access to clients, including but not limited to phoning clients directly, other than at the request of a provider or staff member  Market their services to clients while on the job, including but not limited to, arranging services or appointments for clients in order to create business for themselves; or  Transport clients for any business, including social service, L&I or medical appointments 62
  • 63. Scope of Practice:  Avoid Over-Familiarity with Patients and Family Members  Do not give out your contact information  Share only general and non-controversial information with patients/clients  Avoid making personal comments unless positive and non-intimate  Do not encourage chatter about relationships, troubles, opinions  Avoid Acting out of Role  Do not comment on the patient’s health care situation AT ALL  Do not make ANY specific recommendation to the patient/client regarding his healthcare or services  Fold any request for opinion or recommendation from the patient or client back around into getting the patient/client to ask the staff 63
  • 64. Scope of Practice: Ethical Actions  Use a polite script to decline discussing personal subjects  Deflect or distract the individual from disclosing info about his care or services, or remove yourself from the vicinity politely  Educate individuals about boundaries of the interpreter role in general 64
  • 65. Script to politely decline discussing personal subjects  I prefer to keep my personal life separate from my work, so I can focus on doing a great job for you.  Today I’m here for YOU, to help you communicate your concerns and understand your provider’s recommendations fully.  I’m here to help you communicate with your provider. We should stay focused on you and your immediate concerns. 65
  • 66. Scope of Practice-Quiz  Interpreter sits on far side of waiting room. Patient’s wife comes over to interpreter and says that she is worried because her husband has had chest pain but will only be telling the nurse that he has indigestion. The interpreter should: A) Say that chest pain is a bad sign, the patient probably has a heart condition, and the wife should speak up to the nurse B) Say that any concern from the patient or the family is important for the care team to know about, and would the wife like the interpreter to accompany her now to talk to the nurse? C) Go over to the patient and tell him that he is playing with fire to keep important symptoms hidden, and that he should go right now to talk to the nurse, as every minute might be his last D) Tell the wife that if it is the patient’s time to die, God will take him, otherwise he will be fine 66
  • 67. Scope of Practice-Quiz  Interpreter sits on far side of waiting room. Patient’s wife comes over to interpreter and says that she is worried because her husband has had chest pain but will only be telling the nurse that he has indigestion. The interpreter should: A) Say that chest pain is a bad sign, the patient probably has a heart condition, and the wife should speak up to the nurse B) Say that any concern from the patient or the family is important for the care team to know about, and would the wife like the interpreter to accompany her now to talk to the nurse? C) Go over to the patient and tell him that he is playing with fire to keep important symptoms hidden, and that he should go right now to talk to the nurse, as every minute might be his last D) Tell the wife that if it is the patient’s time to die, God will take him, otherwise he will be fine 67
  • 68. Reporting Obstacles to Practice: Ethical Requirement  Withdraw from encounter if you have:  doubts about your own competence to manage the  lack of knowledge,  emotional involvement,  impairment  Unease about encounter violating code of ethics in some  might make client or patient uncomfortable;  client or patient is too close (family or friend) to interpreter, thus an ethics violation 68
  • 69. Reporting Obstacles: Quiz  Interpreter arrives at encounter in the clinic to find that the patient is a teenage girl who has been raped in an incest situation. As the provider begins to solicit details, the interpreter feels herself getting more and more tense, angry, upset and distracted, even fearful. Why? She and her older sister experienced the same thing  The interpreter should: A) Realize that she will not be able to interpret properly for patient B) Realize that it is critical to not add any pain or embarrassment to the patient by stating that she cannot interpret because of the topic C) Run out of the room and pretend that she has food poisoning D) Quietly use the Interpreter Voice to say to both patient and provider that she must excuse herself because she is not sure that she can do a good enough job of interpreting, but she will stay until another method of language support is arranged for. She says that what the patient and doctor will talk about is very important, and she wants to make sure that they have the services of an interpreter who can properly and professionally perform E) All but one of the above 69
  • 70. Reporting Obstacles: Quiz  Interpreter arrives at encounter in the clinic to find that the patient is a teenage girl who has been raped in an incest situation. As the provider begins to solicit details, the interpreter feels herself getting more and more tense, angry, upset and distracted, even fearful. Why? She and her older sister experienced the same thing  The interpreter should: A) Realize that she will not be able to interpret properly for patient B) Realize that it is critical to not add any pain or embarrassment to the patient by stating that she cannot interpret because of the topic C) Run out of the room and pretend that she has food poisoning D) Quietly use the Interpreter Voice to say to both patient and provider that she must excuse herself because she is not sure that she can do a good enough job of interpreting, but she will stay until another method of language support is arranged for. She says that what the patient and doctor will talk about is very important, and she wants to make sure that they have the services of an interpreter who can properly and professionally perform E) All but C 70
  • 71. Professional Development-Ethical Actions  Assess your knowledge  Language (both source and target languages)  Technical  Cultural  Community  Healthcare System  Address your knowledge gaps via TRAINING/CONTINUING EDUCATION  Assess your skills  Memory  Ability to maintain Register  Managing the flow  Use of diplomatic scripts  Address your skills gaps 71
  • 72. Professional Development-Ethical Actions  Assess your knowledge (technical understanding, language, culture, community, system), then  Address your knowledge gaps, then  Assess your skills, then  Address your skill gaps  Training; CE; professional relationships and consultation 72
  • 73. Professional Development-Quiz  The interpreter receives a steady stream of solicitations from agencies to interpret. No one ever checks her specific knowledge of encounter topics, assuming that she is interpreting accurately and completely. She should: A) Spend all of her time doing assignments, as that helps the most people in the community. B) Keep a log of every instance in which she had trouble understanding the content of a conversation. C) Stay away from interpreter training classes because that would let everyone know that she had more to learn. D) Target the areas in which she has identified deficits in her knowledge or skill set, and take classes in those subjects. E) Two of the above. Which two? 73
  • 74. Professional Development-Quiz  The interpreter receives a steady stream of solicitations from agencies to interpret. No one ever checks her specific knowledge of encounter topics, assuming that she is interpreting accurately and completely. She should: A) Spend all of her time doing assignments, as that helps the most people in the community. B) Keep a log of every instance in which she had trouble understanding the content of a conversation. C) Stay away from interpreter training classes because that would let everyone know that she had more to learn. D) Target the areas in which she has identified deficits in her knowledge or skill set, and take classes in those subjects. E) B and D 74
  • 75. Review  The Code of Ethics governs role boundaries and expectations for behavior and performance of all interpreters:  preparation for the work, initially and continuously  accountability for a good product being delivered (completeness, accuracy)  cultural awareness and support of patient and client values and needs  protection of patients and clients from financial predation  protection of interpreter and patient/client from inappropriate mixing of social and professional interactions  protection of patient/client from control by interpreters 75
  • 76. Questions? Contact Information for:  Eliana Lobo Lobo Language Access InterpreterTrainer@outlook.com  Jazmin Manjarrez OSTI Vice President classymexi55@gmail.com 76

Editor's Notes

  1. L
  2. E
  3. L
  4. E
  5. L
  6. L
  7. L
  8. L
  9. E
  10. L
  11. E
  12. L
  13. E
  14. L
  15. E
  16. E